Hurst Readiness Exam 4

अब Quizwiz के साथ अपने होमवर्क और परीक्षाओं को एस करें!

A 13 year old, found unresponsive in the park, is brought into the emergency department. The nurse sees a medical alert bracelet stating "Diabetic", and notes a fruity smell to the breath. There are no family members available to obtain consent for treatment and an attempt to call them has been unsuccessful. What action should the nurse take? You answered this question Incorrectly 1. Obtain consent from the social worker on duty in the emergency department. 2. Begin treatment by inserting two large bore IVs of Normal Saline. 3. Give glucogon IM and wait for the arrival of a parent to consent to further treatment. 4. Withhold treatment until a parent arrives to the emergency department.

Rationale 2. Correct: In emergencies, if it is impossible to obtain consent from the client or an authorized person, a health care provider may perform a procedure required to benefit the client or save a life without liability for failure to obtain consent. In such cases the law assumes that the client would wish to be treated. Begin treatment for diabetic ketoacidosis (DKA). 1. Incorrect: Consent for a minor is not needed in the event of an emergency. The social worker does not give consent in this situation. 3. Incorrect: This client is exhibiting signs of DKA, so glucagon is not needed. Emergency treatment can be provided without parental consent. 4. Incorrect: Consent for a minor is not needed in the event of an emergency. This is an emergency, so begin treatment for DKA.

Which home routines help reduce the risk of skin damage in a client with impaired sensation? You answered this question Correctly 1. Using a hot water bottle to help warm up when first going to bed. 2. Hot water heater set at a temperature of 140 °F (60 °C) 3. Open flame heaters in the living areas of the home. 4. Testing the water with the back of the wrist and forearm before getting in the shower. 5. Check condition of all equipment used in the home.

Rationale 4. & 5. Correct: This practice actually is a safeguard for skin damage. Equipment should be checked for cracked cord, frayed wire, and exposed heating component. 1. Incorrect: This client should not use a hot water bottle at bedtime because of impaired sensation. The client may not be able to determine that the water bottle is too hot. 2. Incorrect: The water heater should be set at temperatures of about 120 °F (48.88 °C) to decrease likelihood of skin burns. 3. Incorrect: If clients have impaired sensation, burning of the skin is a risk with the use of open flame heaters. The client may sit too close to the heater and be unaware of a burn.

The nurse is caring for a client in an outpatient clinic. The client is being treated with warfarin for prevention of a stroke due to atrial fibrillation. The international normalized ratio (INR) was noted to be 4.6. What should the nurse do? You answered this question Incorrectly 1. Inform the primary healthcare provider immediately. 2. Instruct the client to continue medication as ordered. 3. Inform the client to watch for signs of bleeding. 4. Inform the client to return to the clinic per routine monitoring schedule. 5. Take no action as this value is within target range.

RationaleStrategies 1. & 3. Correct: The primary healthcare provider should be notified. The value of 4 is above the usual target range of 2-3. The client has a potential for decreased clotting and bleeding. The client should be told to watch for signs of bleeding. 2. Incorrect: The medication dosage is likely to be reduced. 4. Incorrect: The client should not leave the clinic until the primary healthcare provider has been notified. Further action is indicated and may include changing the usual warfarin dosage. 5. Incorrect: The normal range for a INR is 2-3. When a client is prescribed warfarin, the INR should increase to a therapeutic target range. The value of 4.6 is greater than the usual target range.

Which assignment would be most appropriate for the charge nurse to assign to the LPN/VN in the Labor, Delivery, Recovery and Postpartum Unit (LDRP)? You answered this question Correctly 1. Primipara needing assistance with breastfeeding. 2. Multipara reporting a headache and epigastric discomfort. 3. Primipara who is two days post op cesarean section. 4. Primipara who is preeclamptic in active labor. 5. Multipara post op cesarean section with a PCA pump.

RationaleStrategies 1. & 3. Correct: These are stable clients whose care is within the scope of practice of an LPN/VN. 2. Incorrect: This client is high risk because she is exhibiting symptoms of postpartum onset preeclampsia. 4. Incorrect: This client is considered to be at high risk since she is in labor and exhibiting symptoms of preeclampsia. This is an unstable client whose care is not within the scope of the LPN. 5. Incorrect: This client has an IV narcotic infusing which cannot be assigned to an LPN. IV narcotics are within the scope of the RN only.

The nurse is caring for a client with hyperparathyroidism. The nurse will monitor the client for which complications? You answered this question Correctly 1. Kidney stones 2. Diarrhea 3. Osteoporosis 4. Tetany 5. Fluid volume deficit

RationaleStrategies 1. & 3. Correct: Yes, because too much calcium in the blood equals too much calcium in the urine and increased risk of kidney stones. Increased parathyroid hormone (PTH) is pulling the calcium from the bones, leaving them weak. 2. Incorrect: A clinical manifestation of hyperpararthyroidism is constipation. Diarrhea is not a clinical manifestation of hyperparathyroidism. 4. Incorrect: Tetany is a clinical manifestation of hypoparathyroidism. 5. Incorrect: Fluid volume deficit (FVD) is not a clinical manifestation of hyperparathyroidism.

What is the best instruction the nurse should provide when administering acetylsalicylic acid 81 mg to a client experiencing severe, crushing chest pain radiating up the left jaw? You answered this question Correctly 1. Chew the acetylsalicylic acid prior to swallowing. 2. Place the acetylsalicylic acid under the tongue so that it can dissolve. 3. Swallow the acetylsalicylic acid tablet. 4. Insert the acetylsalicylic acid between the cheek and gum for greater absorption.

RationaleStrategies 1. Correct: Acetylsalicylic acid has been shown to decrease mortality and re-infarction rates after MI. The fastest way to get the aspirin into the circulatory system is to have the client chew the acetylsalicylic acid prior to swallowing. 2. Incorrect: Nitroglycerin is administered sublingual (SL) or buccal. Initially acetylsalicylic acid is administered by chewing the tablet or swallowing the tablet. 3. Incorrect: If a solid dose pill is prescribed, the pill should be chewed. Faster absorption is obtained from chewing, rather than swallowing acetylsalicylic acid. 4. Incorrect: Nitroglycerin is administered SL or buccal. Initially acetylsalicylic acid would be chewed to increase the absorption rate.

The nurse is caring for a client suffering from major depression. The client spends all day in bed. Which nursing action is appropriate? You answered this question Incorrectly 1. Frequently initiate contact with client. 2. Frequently round at regular intervals. 3. Patiently wait for the client to come out of the room. 4. Question the client about reason for not getting out of the bed.

RationaleStrategies 1. Correct: Be accepting of, and spend time with the client. The client may exhibit pessimism and negativism. The nurse should focus on strengths and accomplishments, and minimize failures. 2. Incorrect: During depressive episodes the client's judgment and problem solving skills are decreased. The nurse should round at frequent, irregular intervals so that the client does not know when to expect the nurse and then can attempt suicide. 3. Incorrect: The nurse should seek out the client. The client may exhibit psycho motor retardation such as lethargy and fatigue. The depressed client is not likely to come looking for someone. 4. Incorrect: Do not confront the client about lack of activity. This will not promote trust. The client may not know why. The client may also have decreased ability to comprehend the question and formulate an answer.

The unit charge nurse is responsible for reporting all healthcare associated infections. Which client condition needs to be reported? You answered this question Correctly 1. A client diagnosed with Clostridium Difficile while receiving intravenous (IV) antibiotics. 2. A client admitted with Methicillin-Resistant Staphylococcus aureus (MRSA) in a wound. 3. A client with ulcerative colitis exhibiting diarrhea. 4. A client with a fever of 99.1º F (37.2° C) two days post gastrectomy.

RationaleStrategies 1. Correct: Clostridium Difficile is a spore forming bacterium that has significant healthcare associated infections (HAI) potential. Clients with intravenous catheters are at a higher risk for HAI. 2. Incorrect: This client was admitted with MRSA already present which indicates that this is a community acquired infection. The client did not acquire a healthcare associated infection. 3. Incorrect: Clients with ulcerative colitis have diarrhea. Diarrhea in this instance does not indicate a possible healthcare associated infection. 4. Incorrect: Low grade fever may occur after surgery. The temperature of 99.1°​ F (37.2° C) does not indicate a HAI at this time.

The nurse observes a client at a follow-up appointment using correct cane walking technique but losing balance each time the quad cane is lifted off of the floor. The client reports a history of recent falls. What is the best action for the nurse to take? You answered this question Incorrectly 1. Inform the primary healthcare provider of the observations made regarding quad cane use, and imbalance assessment. 2. Inform client that there are only a few assistive devices available to help with ambulation. 3. Instruct the client on proper quad cane use. 4. Notify the primary healthcare provider after consulting with the neighbor.

RationaleStrategies 1. Correct: Inform the primary healthcare provider of the observations made regarding the quad cane. The client can not maintain proper balance with the assistance of one sided support. This client would most likely benefit from a walker. 2. Incorrect: The word "only" in the answer indicates that the choices for the client are limited. There are other interventions to assist with balance control. Therefore, this is not correct information or action for the nurse to take. 3. Incorrect: The client was observed using proper technique. Instruction on proper technique is not necessary. The nurse needs to identify the client problem which is imbalance. 4. Incorrect: Discussing the nurse's observation of the client with a neighbor would be a breach of client confidentiality. The Health Insurance Portability and Accountability Act (HIPAA) protects the individual's personal health information.

A client is seen in the clinic for recurrent unexplained, vague stomach pain over the past 5 years. EGD, colonoscopy, gallbladder ultrasound, and lab results have revealed no physical reason for the symptoms. The client tells the nurse, "The doctor thinks the pain in my stomach is psychosomatic. But the pain is so bad some times that I can't function!" What is the nurse's most appropriate response? You answered this question Correctly 1. "The pain you feel is real." 2. "The primary healthcare provider is right. Your pain is not real." 3. "Let me get you an appointment with the psychiatrist." 4. "Don't worry. Everything will be ok."

RationaleStrategies 1. Correct: Pain is real even if it is psychological pain. The client is experiencing anxiety, or stress through stomach pain. The nurse should use therapeutic communication technique that is client centered and empowers the client. 2. Incorrect: This is a example of nontherapeutic communication. The response is confrontational and does not address how the client feels. 3. Incorrect: This nontherapeutic communication of changing the subject ignores the client's feelings. This action invalidates the client. 4. Incorrect: This is a nontherapeutic communication technique, because the response is trite, with false reassurance. The nurse can not know if everything will be ok for the client.

A homebound client lives alone, has a history of poorly controlled diabetes, and has an open wound on the left heel. The home health nurse is concerned about the client's condition and the possible need for a referral. Which intervention should the nurse initiate for this client? You answered this question Incorrectly 1. Ask the primary healthcare provider to prescribe a diabetes educator consult. 2. Increase home health visits to monitor the healing process of the open wound. 3. Suggest nursing home placement to the family until wound has healed. 4. Suggest that the client's family hire sitters to assist with hygiene care.

RationaleStrategies 1. Correct: Referrals to appropriate agencies or departments are often made by the home care nurse. Client needs must be met in the most efficient way while utilizing appropriate expertise. This client has poorly controlled diabetes resulting in a wound. A diabetes educator can help develop a plan to prevent further complications of diabetes. 2. Incorrect: The home care nurse has identified an immediate need which must be met. Increasing the number of visits may be part of the nurse educators plan but the total care of this client needs to be assessed. Healing is not the primary problem, poorly controlled diabetes is the problem. 3. Incorrect: Nursing home placement may or may not be needed. Other disciplines may need to be involved in the care of the client before suggesting alternate placement. Remember to fix the problem and the nurse educator will fix the poorly controlled diabetes. 4. Incorrect: Bringing in sitters for hygiene needs will not fix the problem. The problem is poorly controlled diabetes and this poor control has resulted in an open wound.

A new mother brings her infant to the clinic for a well-baby checkup. While at the clinic, the mother asks the nurse if there are any reasons why her infant should not have the measles, mumps, rubella (MMR) vaccine. The nurse's response is based on evidence that the MMR vaccine is contraindicated under which condition? You answered this question Correctly 1. A known allergy to gelatin. 2. A family history of autism. 3. In infants with diarrhea. 4. A known allergy to sulfonamides.

RationaleStrategies 1. Correct: The MMR vaccine is grown using chicken embryos and manufactured with the use of gelatin. Known allergies to gelatin would be a contraindication for administration. 2. Incorrect: The Centers for Disease Control does not recognize a link between the administration of the MMR vaccine and the development of autism. 3. Incorrect: Diarrhea is not a contraindication specifically for the MMR vaccine. Diarrhea may result in hypovolumia and electrolyte imbalance which need to be addressed. 4. Incorrect: Sulfonamides are not used in the development of the MMR vaccine. Neomycin is used in the development of the MMR vaccine. Neomycin is the only antibiotic allergy that would contraindicate the administration of the MMR vaccine.

The nurse is caring for an adolescent client diagnosed with depression. The client is prescribed fluoxetine. What is the best response by the nurse when the client says, "What will this medicine do to me?" You answered this question Correctly 1. It will increase the level of serotonin in the brain. 2. It will decrease the production of noradrenaline. 3. It will lower your level of the brain hormone norepinephrine. 4. It will balance blood glucose and dopamine levels in your head.

RationaleStrategies 1. Correct: The action of fluoxetine is to increase the level of serotonin in the central nervous system. There is a correlation between a low level of serotonin and depression. The action of the drug should be explained to the adolescent in a manner that will be understood. 2. Incorrect: Fluoxetine does not selectively decrease the production of noradrenaline in the brain. 3. Incorrect: This action does not relate to fluoxetine. Fluoxetine may actually increase the level of norepinephrine in the brain. 4. Incorrect: This action does not relate to fluoxetine. Fluoxetine does not effect the balance of blood glucose and dopamine in the brain.

While performing a vaginal examination on a client in labor, the nurse feels soft, squishy tissue instead of a head. What conclusion should the nurse make based on this assessment finding? You answered this question Correctly 1. Breech presentation 2. Edema of cervix 3. Closed cervix 4. Bulging membranes

RationaleStrategies 1. Correct: The nurse is palpating the buttocks of the fetus. The buttocks would be assessed as soft, squishy tissue. This is evidence of a breech presentation. 2. Incorrect: The nurse is assessing soft, squishy tissue. The edema of the cervix is noted by the thickened edge of cervix. 3. Incorrect: Would not be able to feel presenting part with a closed cervix. When the cervix is closed, the nurse is unable to assess any of the fetus's presenting parts. 4. Incorrect: Bulging membranes will be taunt, not soft and squishy.

A client tells the nurse, "I am dying from cancer. I have told my primary healthcare provider that I do not want to be revived if my heart stops beating or I stop breathing." What action should the nurse take first to assure that the client's request is respected? You answered this question Correctly 1. Ensure a do-not-resuscitate prescription has been provided. 2. Report client wishes during the end-of-shift report. 3. Have the client sign an advanced directive. 4. Ask the client who holds the durable power of attorney for health care decisions.

RationaleStrategies 1. Correct: The nurse should check the medical record for a DNR order. By law, a person who does not have a do-not-resuscitate (DNR) prescription, must be provided CPR in the event of a cardiac/respiratory arrest. This action will ensure the client's end-of-life wishes have been communicated and will honor the client's wishes. 2. Incorrect: It is appropriate to report the client's end-of-life wishes to other care givers, but not before ensuring a DNR order is in place. 3. Incorrect: If the client has advance directives, a copy should be placed in the medical record. However, a DNR prescription must also be in place to ensure the client is not resuscitated. 4. Incorrect: The client's request can be initiated by notifying the primary healthcare provider. It would be helpful for the client to have a durable power of attorney.

A nurse from the neonatal unit is transferred to the adult medical-surgical unit. Which client should the charge nurse assign to the neonatal nurse? You answered this question Incorrectly 1. Undergoing surgery for placement of a central venous catheter. 2. Diagnosed with leukemia, hospitalized for induction of high-dose chemotherapy. 3. Receiving IV heparin for left leg thrombosis. 4. Admitted with a cerebrovascular accident.

RationaleStrategies 1. Correct: This is the most stable client to give to the nurse who was transferred from the neonatal unit. A neonatal nurse cares for central lines daily in this specialty area and can transfer this knowledge to the adult client. 2. Incorrect: This is not a good client for a neonatal nurse because knowledge of lab values, chemotherapy precautions, protective isolation and chemotherapy drugs is required for the nurse in order to care for this client. 3. Incorrect: This is not the best client for a neonatal nurse because thrombosis problems are not commonly seen in the nursery. Monitoring clotting factors and being aware of signs and symptoms of pulmonary emboli are essential for safe care of this client. 4. Incorrect: This client is very unstable and requires skilled observation and assessment using the Glasgow Scale. This level of assessment is not utilized in a neonatal unit.

Which observations by the nurse indicate that a mother is protecting her two day old female newborn from infection? You answered this question Incorrectly 1. Cleans perineum from front to back after newborn soils diaper. 2. Makes certain the umbilical cord remains dry with each diaper change. 3. Places the top of the diaper just above the umbilicus. 4. Wraps sterile petroleum gauze around umbilical cord. 5. Submerges newborn in warm water up to the chest for first bath.

RationaleStrategies 1., & 2. Correct: Cleaning from front to back will decrease the risk of infection by reducing the number of microorganisms at the urethral meatus. Keeping the umbilical cord clean and dry will decrease the risk of infection and will allow it to fall off. 3. Incorrect: The top of the diaper should be placed just below the umbilicus to prevent exposure to body waste and moisture. Placing the diaper above the umbilical cord will cause the diaper to rub the umbilicus, which will increase the risk of infection. 4. Incorrect: This would keep the umbilical cord moist and could lead to infection. Also a sterile dressing is not warranted. The umbilical cord needs to be kept dry so it will fall off. 5. Incorrect: The newborn should not be placed in water until after the umbilical cord falls off. Water submersion keeps the cord moist and at risk for infection. The umbilical cord should be kept dry so that it will fall off.

The nurse is planning health promotion strategies for a single parent of young children who is trying to increase personal physical activity level but expresses a lack of time. Which interventions would help the client get more regular physical activity into the day? You answered this question Incorrectly 1. Suggest walking up and down steps at home several times each morning and evening. 2. Suggest parking further away from the grocery store and work. 3. Walk with the children in the evening instead of watching TV with them. 4. Suggest waking one hour earlier in the morning to go to the gym. 5. Suggest walking for 30 minutes with a buddy each afternoon before leaving work.

RationaleStrategies 1., 2. & 3. Correct: This plan will allow the parent to stay home without adding further time demands to the day. Parking farther away is one plan to get more steps into the day without increasing time demands drastically. Walking with the children allows the parent to spend quality time with the children as well as offers them a good example. 4. Incorrect: Being a single parent, this plan would not be feasible. The demands of getting the children out earlier could impact the time schedule for the day in a negative way. 5. Incorrect: This plan would only increase time demands and possibly financial demands if the children have to be cared for by someone else at an extra charge each day.

Which interventions should the nurse include when planning care for a client diagnosed with paranoid personality? 1. Develop a trusting relationship. 2. Be honest when communicating with the client. 3. Encourage the client to participate in group therapy. 4. Encourage the client to clean the day room daily. 5. Give clear explanations of procedures before hand.

RationaleStrategies 1., 2. & 5. Correct: This disorder is characterized by distrust and suspicion towards others. The nurse should use open communication techniques to increase the client's trust in the nurse. Clear explanations of procedures will decrease the anxiety of the client. 3. Incorrect: The client with paranoid personality is reluctant to share personal information with other people. They suspect everyone of causing problems for them. Group therapy would not be appropriate for this client. 4. Incorrect: The client with paranoid personality feels that others are using or exploiting them. The client may perceive that they are being exploited if they clean the dayroom.

The nurse is teaching parents of a school aged child about interventions to keep the child safe. Which interventions would be appropriate to include in the health promotion plan? You answered this question Correctly 1. Encourage bicycle helmet use when riding bikes. 2. Teach children to swim at an early age. 3. Use booster seats until the child is at least 6 years old. 4. Keep firearms in the home locked and unloaded. 5. Teach "stop, drop, and roll" in case clothing catches on fire.

RationaleStrategies 1., 2. 4., & 5. Correct: Role modeling the value of helmet use is an excellent way to encourage the child to use a helmet. Children should learn to swim at early ages. Children may drown in home or neighborhood pools. Children are curious about firearms, which should be safely locked away. Fire safety is important. Being able to extinguish a fire quickly can save a life. 3. Incorrect: Once children outgrow weight and height limits for traditional safety seats, they should travel in belt-positioning booster seats in the back seat until the vehicle seat belt fits properly, typically when they have reached 4'9″ tall and are between 8 and 12 years of age.

What should the nurse teach a pregnant client who comes to the clinic reporting hemorrhoids and constipation? You answered this question Incorrectly 1. Increased rectal pressure from the gravid uterus may result in hemorrhoids. 2. Hormones decrease maternal GI motility, resulting in constipation. 3. The client needs more fiber in the diet. 4. A mild laxative is recommended to alleviate constipation. 5. The client needs to increase fluid intake.

RationaleStrategies 1., 2., 3. & 5. Correct: As pregnancy progresses, the enlarging uterus increases abdominal and rectal pressure. GI motility slows due to hormonal influences. Pregnant clients may benefit significantly from dietary changes including adequate hydration and increased fiber intake. 4. Incorrect: Medications, including laxatives, should not be taken by pregnant women unless prescribed by the primary healthcare provider. If needed, the primary healthcare provider may prescribe a stool softener but rarely a laxative because of possible fluid and electrolyte shifts.

Which interventions should the nurse initiate to lessen acid reflux in a client diagnosed with gastroesophagel reflux disease (GERD)? You answered this question Correctly 1. Provide small, frequent meals. 2. Avoid carbonated beverages. 3. Administer omeprazole as prescribed. 4. Assist with smoking cessation. 5. Place in left lateral position for 2 hours after eating.

RationaleStrategies 1., 2., 3., & 4. Correct: All of these actions are correct to help alleviate dyspepsia. When a client has GERD, the stomach's contents reflux into the esophagus. Small frequent meals will decrease possible reflux by decreasing the stomach content. Smoking can relax the lower muscle of the esophagus. Drinking a carbonated drink may cause the stomach to expand. Both smoking and drinking a carbonated drink increase the potential of reflux. The action of omeprazole is to reduce the acid that is produced in the stomach. 5. Incorrect: The client should be positioned with the head of the bed (HOB) elevated for 2-3 hours after eating. This position will decrease the potential for esophageal reflux.

A nurse is planning to provide an education class on preconception health care to a group of young women wishing to become pregnant. What points should the nurse include in this class? You answered this question Correctly 1. Attain a healthy weight. 2. Make sure immunizations are up to date. 3. Avoid drinking alcohol. 4. Learn family health history. 5. Maintain folic acid intake at 200 micrograms/day.

RationaleStrategies 1., 2., 3., & 4. Correct: All of these actions are needed to promote the birth of a healthy baby. A preconception care visit or class can help women take steps for a safe and healthy pregnancy before they get pregnant. 5. Incorrect: Folic acid intake should be 400 micrograms per day in order to reduce neural tube defects by 70%.

A client arrives at the emergency department after sustaining partial and full-thickness burns over the anterior neck, chest, and right arm. Which interventions will the nurse initiate? You answered this question Correctly 1. Administer oxygen 2. Start two intravenous lines 3. Remove necklace 4. Elevate right arm 5. Debride wounds

RationaleStrategies 1., 2., 3., & 4. Correct: Burns over the anterior neck and chest mean that the client is likely to have inhalation burns, putting him/her at high risk for impaired gas exchange. The inhalation will cause edema of the airway. It goes back to Maslow's Hierarchy of Needs. Administer oxygen and start two IVs so that fluid resuscitation can begin. Metal continues to burn and swelling will occur, so remove the necklace or any jewelry. Elevate the arm to decrease swelling. 5. Incorrect: Debridement of the wound occurs in the acute/intermediate phase. This phase starts 48-72 hours after the burn accident has happened.

A nurse has just inserted an indwelling urinary catheter in a client scheduled for surgery. What should the nurse document? You answered this question Correctly 1. Perineal skin assessment 2. Client teaching 3. Color of urine 4. Date and time of insertion 5. Type catheter inserted 6. Infusing rate of IV fluid

RationaleStrategies 1., 2., 3., 4., & 5. Correct: Perineal skin assessment should be assessed prior to insertion of the indwelling catheter. The following documentation is appropriate after inserting an indwelling catheter: Client teaching, color of the urine, date and time of the insertion and the type of catheter inserted. 6. Incorrect: Documenting the IV rate is not relevant to inserting the catheter. The infusion rate may need to be documented, but the question is asking about the documentation of the insertion of the indwelling catheter.

A quality assurance (QA) manager plans to evaluate performance improvement regarding the implementation of fall precautions for at risk clients. What steps should the QA manager include in this evaluation? You answered this question Incorrectly 1. Chart review for fall precaution documentation. 2. Direct observation of unit staff. 3. Poll staff to identify what fall precautions are implemented for at risk clients. 4. Identify at risk clients on unit. 5. Make unannounced visits to the unit for evaluating staff performance.

RationaleStrategies 1., 2., 4 & 5. Correct: The QA manager is responsible for evaluating performance improvement plans to ensure that staff are providing appropriate care. The QA manager can do chart reviews to see if staff are documenting fall precaution for a client. Direct observation of unit staff will let the QA manager know if staff are performing proper precautions while caring for clients. The first step is to identify what clients are at risk for falls and then see if the staff have identified these clients as at risk as well. Monitoring should be at unpredictable intervals, so staff do not comply just for a scheduled evaluation. 3. Incorrect: Asking the staff does not ensure that they follow through.

The nurse is teaching the client about benzodiazepines. Which comments by the client indicate adequate understanding of the drug effects/side effects? You answered this question Incorrectly 1. I should not drive my car until I see how the medication affects me. 2. I can expect my reaction time to be slowed in the beginning. 3. I may need to double the dose if I continue to be anxious. 4. I must be careful to take the medication for a limited time. 5. There is a risk for dependence on this medication.

RationaleStrategies 1., 2., 4. & 5. Correct: Benzodiazepines slow reaction time and may affect general alertness. The client should not operate machinery until effects of the medication are observed, and client can drive safely. Benzodiazepine medications are usually prescribed for short periods of time. Benzodiazepines are frequently abused. Clients develop tolerance and dependence on the drugs. 3. Incorrect: The client should not self-regulate dosage. There is a potential for tolerance and dependence to develop. Dosage should be monitored carefully by the primary healthcare provider.

The home care nurse visits a client who has moderate cognitive impairment and whose family provides care for the client. Which suggestions would be helpful for this family to reduce the risk of injury? You answered this question Correctly 1. Suggest that the family lock up medications and poisons and keep the keys. 2. Encourage the family to place locks high on the door frame to make it difficult for the client to leave. 3. Suggest that the family talk with the client weekly about safety issues around the house. 4. Suggest that the family remove knobs from stove when not in use. 5. Keep fire extinguishers present and in working order.

RationaleStrategies 1., 2., 4. & 5. Correct: Clients with cognitive impairment may forget that they have taken their medicines and take them again. They may also confuse harmful substances with other substances. Locks in places that are not normally expected will make it more difficult for the client with a cognitive impairment to find and open. This is especially useful if the client wanders. The client may turn the stove on and be burned or cause a fire. If the knobs are removed, the home is safer for everyone. Fires are a hazard for people with cognitive impairment; therefore, the presence of a working fire extinguisher could prevent damage from a fire. 3. Incorrect: A client with moderate cognitive impairment will need to be continually supervised to decrease their risk of injury. The retention of information this client has is too short for weekly discussions.

A case manager is assessing an unresponsive client diagnosed with terminal hepatic encephalopathy to determine equipment needs upon discharge to home for hospice care. Which equipment should the case manager obtain for this client? You answered this question Correctly 1. Alternating pressure mattress 2. Hospital bed 3. Walker 4. Suction equipment 5. Oxygen

RationaleStrategies 1., 2., 4., & 5. Correct: An alternating pressure mattress will help to prevent pressure ulcers. A hospital bed is needed so that the head of the client's bed can be elevated to 30 degrees to ease respirations and decrease the work of breathing. The unresponsive client may need suction equipment for suctioning if unable to clear secretions from the oropharynx. The client at the end stages of liver disease will be hypoxemic, so oxygen therapy is provided. 3. Incorrect: The unresponsive client will not need a walker.

What statements by a client diagnosed with a hiatal hernia would indicate to the nurse that the discharge teaching was effective? You answered this question Correctly 1. "I should eat six small meals a day." 2. "Sitting up for an hour after I eat will decrease symptoms." 3. "Eating a grapefruit for breakfast will help digest the rest of my food." 4. "Ten inch blocks need to be placed under the head of my bed." 5. "I will get assistance for lifting heavy objects." 6. "I will avoid using laxatives."

RationaleStrategies 1., 2., 4., & 5. Correct: Clients with a hiatal hernia should eat small frequent meals, because large meals cause them to be symptomatic with heartburn and other symptoms. Sitting up after eating will keep the stomach down as much as possible. If they lie down, the stomach will go upward and cause regurgitation, heartburn, nausea, and fullness. Placing blocks under the bed also helps keep the stomach downward and reduces symptoms when the client sleeps. One of the major causes and aggravating actions for a hiatal hernia is straining. Therefore, the clients do not need to be lifting heavy objects. 3. Incorrect: Since grapefruits are acidic, they can increase the amount of acid backing up into the esophagus. Eating grapefruits should be avoided. 6. Incorrect: Straining should be avoided so use of laxatives may be advised. Straining to have a bowel movement will cause increased abdominal pressure which may cause pressure on the hiatal hernia.

An elderly client arrives in the emergency department (ED) after a fall. What assessment findings would lead the nurse to suspect that the client has a fractured right hip? You answered this question Incorrectly 1. Severe pain in the right hip and groin. 2. Inability to bear weight on the right leg. 3. Right leg slightly longer in length than the left leg. 4. External rotation of right lower leg. 5. Bruising and swelling around the right hip.

RationaleStrategies 1., 2., 4., & 5. Correct: Pain in the affected hip, often severe, is one of the main signs of a hip fracture. This pain may radiate to the groin area. The pain and bone injury generally prevent the client from being able to bear weight on the affected leg. The client will often assume a position in which the leg on the injured side is held in a still and externally rotated position (the foot and knee turns outward). Discoloration and swelling can be an indication of a hip fracture in some clients. 3. Incorrect: A client who has a hip fracture often appears to have shortening of the extremity on the affected side. This is a result of the location of the break and the positioning of the body in response to the injury and pain.

Which tasks would be appropriate for the nurse to assign to an LPN/VN? You answered this question Incorrectly 1. Changing a colostomy bag. 2. Administer antibiotic via intravenous piggyback (IVPB). 3. Teach insulin self administration to a diabetic client. 4. Administer IV pain medication to a two day post op client. 5. Check for urinary retention. 6. Remove wound sutures.

RationaleStrategies 1., 2., 5., & 6. Correct. These tasks are within the PNs practice scope. The PN can change a colostomy bag, administer antibiotics by IVPB, monitor for urinary retention and remove wound sutures. 3. Incorrect: The RN is responsible for teaching. The PN can reinforce teaching once taught by the RN. 4. Incorrect: The RN must give IV pain meds to clients. The PN can monitor the effectiveness of the medication after given by the RN and can report any problems if necessary.

The nurse is providing discharge education to a client after a concussion. What should the nurse emphasize to report to the primary healthcare provider? You answered this question Correctly 1. Difficulty waking up 2. Headache (3/10 on the pain scale) 3. Blurry vision 4. Achy feeling all over 5. Vomiting

RationaleStrategies 1., 3. & 5. Correct: Increased intracranial pressure (ICP) is a result of increased pressure around the brain or blood in the brain. These are signs of increasing intracranial pressure (post-concussion syndrome). This is a medical emergency, and the PHP should be notified immediately. 2. Incorrect: A headache of 3/10 on the pain scale does not warrant notifying the primary healthcare provider. The primary healthcare provider should be notified if the pain intensity increases. 4. Incorrect: This is not related. This is not a symptom of increased ICP.

A nurse is planning to teach a group of men about their sildenafil prescription. What information should the nurse include? You answered this question Incorrectly 1. Notify primary healthcare provider if prescribed an alpha-adrenergic blocker. 2. This medication protects against sexually transmitted diseases. 3. Sildenafil should be taken only once per day if needed. 4. This medication is most effective if taken with grapefruit juice. 5. The most common side effects are flushing, headache, and dyspepsia.

RationaleStrategies 1., 3., & 5. Correct: Alpha-adrenergic blockers action is to dilate small muscles. The action of sildenafil is also to dilate specific small muscles. The combination of these medications can cause a hypotension event. Sildenafil is usually taken only when needed, 30 minutes to 1 hour before sexual activity. Do not take sildenafil more than once per day. The most common side effects are flushing, headache, and dyspepsia. 2. Incorrect: This medication does not protect against sexually transmitted diseases. Safe sex practices should be followed, such as use of latex condoms. 4. Incorrect: Grapefruit and grapefruit juice may interact with sildenafil and lead to unwanted side effects. Avoid the use of grapefruit products while taking sildenafil.

The public health nurse is planning to participate in local forums regarding the placement of a factory that is known to produce pollution through discharge of chemical by-products into the air. What actions demonstrate ethical nursing practice in the public health arena? You answered this question Correctly 1. Speaking up for the underrepresented, such as the poor and uneducated persons. 2. Encouraging community leaders to accept placement of the factory. 3. Requesting that forums be held throughout the community at various times of the day or evening. 4. Asking for information regarding the health status of people in other factory locations. 5. Requesting information from individuals in areas where the factories are currently located.

RationaleStrategies 1., 3., 4. & 5. Correct: Many times factories are placed in communities where people are not aware of the hazards. The underrepresented and poor need the nurse as their advocate. Forums encourage wider participation of all community members and give the community more information about the consequences of the pollution. The public health nurse advocates for the health of the entire community. Individuals in the communities where factories are located could give first-hand information about health or other issues related to the factory placement. Printed reports, depending on the source, may contain false information. 2. Incorrect: More information is needed at this time. Placement should be determined by what is best for the community as a whole. The nurse is interested in protecting the public's health.

The primary healthcare provider is preparing to drain a large abdominal abscess. The client has dementia and moves about on the bed frequently. Which personal protective equipment (PPE) should the nurse wear while holding the client for the procedure? You answered this question Incorrectly 1. Face shield 2. Sterile Gloves 3. Gown 4. Mask 5. Regular exam gloves

RationaleStrategies 1., 3., 4. & 5. Correct: The nurse should implement transmission based contact precautions. During drainage of an abscess, the nurse may come into direct and indirect contact of the contaminated body fluids. The nurse needs the protection of a gown, mask, face shield, and regular exam gloves. Since the nurse is not directly assisting with the wound care, regular exam gloves are appropriate. 2. Incorrect: Sterile gloves are not necessary since the nurse is holding the client and not directly assisting with the wound care procedure.

A nurse is attempting to develop trust with a psychiatric client exhibiting concrete thinking. Which nursing intervention would promote trust in this individual? You answered this question Incorrectly 1. Attend an activity with the client who is reluctant to go alone. 2. Allow the client to break an insignificant rule. 3. Consider client preferences when possible in decisions concerning care. 4. Provide a blanket when the client is cold. 5. Provide food when the client is hungry.

RationaleStrategies 1., 3., 4. & 5. Correct: Trust is demonstrated through nursing interventions that convey a sense of warmth and care to the client. These interventions are initiated simply, concretely, and directed toward activities that address the client's basic needs for physiological and psychological safety and security. Concrete thinking focuses thought processes on specifics, rather than generalities, and immediate issues, rather than eventual outcomes. Examples of nursing interventions that would promote trust in an individual who is thinking concretely include such things as: providing a blanket when the client is cold, providing food when the client is hungry, keeping promises, being honest, providing a written, structured schedule of activities, attending activities with the client if he is reluctant to go alone, being consistent in adhering to unit guidelines, and taking the client's preferences, requests, and opinions into consideration when possible in decisions concerning care. 2. Incorrect: The client should be informed of all rules, simply and clearly, with reasons for certain policies and rules. Be consistent and provide written, structured, scheduled activities. Allowing a client to break a rule would not encourage them to think about the outcomes of their actions.

What statements by a new nurse would indicate to the charge nurse an understanding of how to maintain skin integrity for a client on bedrest? You answered this question Correctly 1. "Clients on bedrest should be placed on therapeutic mattresses." 2. "I will assess for the skin every 4 hours." 3. "I will assess the skin using the Braden scale." 4. "A pillow will be placed between the knees when client is side lying." 5. "The incontinent client will be kept clean and dry."

RationaleStrategies 1., 3., 4., & 5. Correct: Clients on bedrest should use a therapeutic bed or mattress. These prevent and treat pressure ulcers by molding to the body to maximize contact, redistributing weight, and reducing pressure. The Braden scale is the most preferred tool to assess risk of developing pressure ulcers. It looks at sensory perception, moisture, activity, mobility, nutrition, and friction/shear. A pillow between the knees can decrease pressure on knees if they were touching. Protect the client from moisture by keeping clean and dry. 2. Incorrect: When a client is on bedrest the skin and subcutaneous tissue can not profuse adequately. Therefore, the skin should be assessed every 2 hours not every 4 hours.

A nurse is instructing a client who had a cesarean birth 2 days ago about adverse signs that should be reported to the primary health care provider. Which signs should the nurse include? You answered this question Correctly 1. Fever greater than 100.4° F (38° C) for 2 or more days. 2. Change in lochia from rubra to serosa. 3. Calves with localized pain, redness, and swelling. 4. Burning with urination. 5. Feeling of apathy toward newborn. 6. Able to provide self care.

RationaleStrategies 1., 3., 4., & 5. Correct: Fever for 2 or more days can indicate infection. Calf pain, redness, and swelling could indicate thrombophlebitis. Burning on urination could indicate urinary tract infection (UTI). Feeling of apathy about the newborn could mean postpartum depression. All should be reported to the primary healthcare provider. 2. Incorrect: You would report change from serosa to rubra lochia. Also, report changes in vaginal discharge with increased amount, large clots, and change to a previous lochia color, such as bright red bleeding and a foul odor. 6. Incorrect: We want the client to be able to provide self care. This does not warrant a call to the primary healthcare provider.

What should the nurse include in the post-op care of a client following the removal of the posterior pituitary gland? You answered this question Correctly 1. Monitor intake and output. 2. Restrict fluids to no more than 1500 ml/day. 3. Weigh daily. 4. Monitor urine specific gravity. 5. Assess the level of consciousness (LOC). 6. Instruct client to avoid blowing the nose forcefully.

RationaleStrategies 1., 3., 4., 5., & 6. Correct: Removal of the pituitary gland can lead to diabetes insipidus (DI) as a result of the reduced production of antidiuretic hormone (ADH). The nurse should monitor I & O closely and watch for an increase in output which would indicate diuresis as part of DI. Daily weights are an important part of monitoring the client's fluid status. Monitoring the urine specific gravity is another good way of assessing the fluid status because, as the urinary output increases, the client's urine is becoming more dilute, which would result in a lower urine specific gravity. If the client's serum volume is decreasing from the excessive diuresis, the client can go into shock. The nurse should monitor for early signs of changes in the level of consciousness. To avoid disrupting the surgical site, the client should not blow the nose forcefully for at least one week post-op. 2. Incorrect: If the client is lacking ADH, the client may begin losing large amounts of fluid volume. Therefore, the fluid intake would need to be increased (not decreased) to avoid dehydration and shock.

A nurse is preparing to conduct a presentation on barriers to therapeutic communication with clients from a culture other than the nurse's culture. Which points should the nurse include in the presentation? You answered this question Incorrectly 1. Lack of knowledge about a client's culture is a major barrier to therapeutic communication. 2. Follow cultural beliefs when caring for all clients of that particular culture. 3. Ethnocentrism facilitates therapeutic communication. 4. Do not touch the client until you know what the cultural belief is about touching. 5. Adapt care to client's cultural needs and preferences.

RationaleStrategies 1., 4. & 5. Correct: Nurses must understand and take into consideration the cultural differences of their clients. Some cultures do not approve of touching or shaking hands. By assessing the client's culture preference, the nurse is able to provide individualized care. 2. Incorrect: Do not stereotype all clients of a certain culture. Ask questions. Allow for individuality. To provide culturally competent care, the nurse must recognize individual preferences within the client's culture. 3. Incorrect: Ethnocentrism is the belief that one's own culture and traditions are better than those of another. It blocks therapeutic communication by allowing the nurse's biases and prejudices to negatively influence the nursing care of the client.

A nurse is planning a health education seminar for a group of females who are age 45-54. What should the nurse recommend be done annually? You answered this question Correctly 1. Chest xray 2. Mammography 3. Influenza vaccine 4. Tuberculous (TB) skin test 5. Colonoscopy

RationaleStrategies 2. & 3. Correct: It is recommended that women age 45-54 should have a mammogram annually. Women 55 or older should have a mammogram every 2 years. The influenza vaccination is recommended annually for persons 6 months and older. 1. Incorrect: Chest x-ray is not done routinely since that exposes the client to low doses of radiation. A chest x ray would be ordered as a diagnostic radiographic examination. 4. Incorrect: Tuberculosis (TB) skin test yearly is required for high risk individuals, such as those working in the healthcare field or in nursing homes or other close-contact areas. This test is not recommended annually at a specific age for low risk individuals. 5. Incorrect: Colonoscopy is recommended for clients beginning at age 50, but not annually (every 10 years with no problems) until the age of 75. A colonoscopy should be performed more frequently if there is a change in bowel habits, obvious or occult blood in the stool or abdominal pain.

The charge nurse is evaluating knowledge of tracheostomy suctioning of a new nurse prior to that procedure being performed. Which statement by the new nurse would indicate to the charge nurse that additional education is needed? You answered this question Correctly 1. "Prior to suctioning, I will hyper-oxygenate the client." 2. "I will instill normal saline bullets to liquefy secretions." 3. "I will allow at least 20 seconds between suctioning passes." 4. "Suctioning will be limited to a maximum of three catheter passes."

RationaleStrategies 2. Correct. The best ways to liquefy secretions are to humidify secretions and hydrate the patient. Do not use normal saline solution or normal saline bullets routinely to loosen tracheal secretions because this practice may reach only limited areas, may flush particles into the lower respiratory tract, may lead to decreased post-suctioning oxygen saturation, increases bacterial colonization, and damages bronchial surfactant. 1. Incorrect. This is a true statement. Prior to suctioning, the client should be hyper-oxygenated. Suctioning a client will cause a decrease in the clients oxygen level. 3. Incorrect. This is a true statement. This allows the client to get oxygen between passes. The nurse should wait at least 20 seconds before suctioning the client again. This allows the oxygenation of the client to increase. 4. Incorrect. This is a true statement. Each session of suctioning should be limited to no more than 3 passes this will allow the client proper oxygenation and to prevent tissue damage during repeated suctioning.

A nurse has responded to the scene of a natural disaster to triage clients. Which client should the nurse triage with a black disaster tag? You answered this question Correctly 1. Traumatic amputation to the left lower leg. 2. 2nd and 3rd degree burns over 75 % of the body. 3. Fracture of the humerus. 4. Blood pressure of 90/40 and lethargic.

RationaleStrategies 2. Correct: 2nd and 3rd degree burns over 60% of the body put the client in the triage category of black- expectant: injuries are extensive and chances of survival are unlikely even with definitive care. Persons in this group should be separated from other casualties but not abandoned. Comfort measures should be provided when possible. 1. Incorrect: This client is tagged red and needs immediate care. Injuries are life-threatening but survivable with minimal intervention. Individuals can move quickly to black-expectant if treatment is delayed. 3. Incorrect: This client is tagged green and placed in the minimal category. Injuries are minor and treatment can be delayed hours to days. Individuals in this group should be moved away from the main triage area. 4. Incorrect: This client is tagged red and is immediate with signs of shock.

The nurse is obtaining a health assessment from the preoperative client scheduled for hip replacement surgery. Which statement by the client would be most important for the nurse to report to the primary healthcare provider? You answered this question Correctly 1. "When I was 8 years old I had chickenpox." 2. "I had rheumatic fever when I was 10 years old." 3. "There is a strong history of gastric cancer in my family." 4. "I have pain in my hip with any movement."

RationaleStrategies 2. Correct: After having rheumatic fever, a client would need to be pre-medicated with antibiotics prior to any surgical or dental procedure to prevent a recurrence. 1. Incorrect: Chicken pox would have no implications on this surgery. 3. Incorrect: Cancer history in the family would have no implications for this surgery. 4. Incorrect: Pain in the hip is likely the reason for the surgery.

Which finding should a nurse expect when assessing a healthy 65 year old client? You answered this question Correctly 1. Anomia 2. Presbyopia 3. BP 156/88 4. Apraxia

RationaleStrategies 2. Correct: As the lens becomes less flexible, the near point of focus gets further away. This condition, presbyopia, usually begins in the 40s. Reading glasses to magnify objects are required. 1. Incorrect: Anomia (cannot name objects) is an early sign of Alzheimer's disease. Anomia is not a normal assessment of a 65 year old client. 3. Incorrect: Blood pressure (BP) reading of 156/88 is not within normal BP range. The normal blood pressure range for the 65 to 79 year old is 140/90 or less. 4. Incorrect: Apraxia means client cannot perform purposeful movement. We would not expect to assess this in a healthy 65 year old.

A client with Crohn's disease develops a fever and symptoms of an infection. The nurse recognizes this complication may occur as a result of which finding? You answered this question Correctly 1. Perianal irritation from frequent diarrhea 2. Fistula formation with an abscess 3. Stricture formation 4. Impaired immunologic response to infectious microorganisms

RationaleStrategies 2. Correct: Clients who suffer from Crohn's disease are at risk for developing fistulas, and an abscess can result from the fistula. 1. Incorrect: Perianal irritation from frequent diarrhea can occur, but irritation does not result in an infection. 3. Incorrect: Stricture formation is a complication, however, these s/s indicate an abscess. 4. Incorrect: Impaired immunologic response is not associated with Crohn's disease.

The nurse is administering the prescribed Mantoux tuberculin skin test to a client. The nurse does not observe the tense blister-like formation at the injection site. Which action should the nurse take? You answered this question Incorrectly 1. Chart the injection site response as the only action. 2. Administer another Mantoux tuberculin skin test at a different site. 3. Circle the area, wait 48 to 72 hours, and assess for a reaction. 4. Call the primary healthcare provider.

RationaleStrategies 2. Correct: If there is not a wheal of at least 6 mm in diameter after the solution is injected , the test should be administered again. The nurse would need to administer another Mantoux tuberculin skin test in another area about 5-6 cm from the original injection site. 1. Incorrect: The Mantoux tuberculin skin test is an intradermal injection. The expected outcome after the injection of the medication is a tense blister-like formation at the injection site. The absence of the tense blister-like formation is an indicator that the injection was given too deep. 3. Incorrect: The Mantoux tuberculin skin test was not administered correctly. A wheal of 5-6 mm did not occur after the injection was given. The test would need to be done again. 4. Incorrect: There is no need to call the primary healthcare provider. The primary healthcare provider prescribed the test. The injection should be administered to create a 5-6 mm wheal.

A client received a leg cast that was applied following fracturing the left femur. What assessment finding would be a priority for the nurse to report to the primary healthcare provider? You answered this question Correctly 1. Reports of a feeling of warmness under the cast after application. 2. Pain not relieved by elevation, cold packs, and pain medication. 3. Reports of itching under the cast not relieved by cool air. 4. Slight swelling of the toes of the affected extremity.

RationaleStrategies 2. Correct: Pain that is disproportionate to the injury, becomes severe, and/or is not relieved by elevation, cold packs, and pain medication could indicate a complication such as compartment syndrome. Failure to detect this could lead to neurovascular damage and possible amputation. 1. Incorrect: Due to the drying process of the cast material, it is normal for the cast to feel warm. The primary healthcare provider would not need to be notified. The warm feeling should subside. 3. Incorrect: A common complaint is itching under the cast. The cast material may cause irritation to the skin. Cool air under the cast may help to relieve this. The primary healthcare provider would not need to be notified at this time. 4. Incorrect: Some swelling is expected initially due to the damage of the tissue around the fracture which may result in dependent swelling of the toes. This compromised circulation should be relieved by elevation. The primary healthcare provider would not need to be notified at this time.

A client with chronic alcoholism has been admitted to the intensive care unit after overdosing on alcohol. Which medication should the nurse prepare to administer? You answered this question Incorrectly 1. Disulfiram 250 mg po daily 2. Thiamine 100 mg IV twice a day 3. Naloxone 0.4 mg IV prn 4. Clonidine TTS patch 2.5 mg per week

RationaleStrategies 2. Correct: Prescribing of thiamine action is to alleviate dehydration, prevent delirium and precaution treatment for vitamin B complex deficiency. Thiamine 50-100 mg IV or IM is indicated twice a day for clients with chronic alcoholism. It is usually given for several days, followed by 10-20 mg once a day until a therapeutic response is obtained. 1. Incorrect: Disulfuram is an aid in the management of selected chronic alcohol clients who want to remain in a state of enforced sobriety so that supportive and psychotherapeutic treatment may be applied. It is not a cure for alcoholism. Without proper motivation and supportive therapy, it is unlikely that it will have any substantive effect on the drinking pattern of the chronic alcoholic. 3. Incorrect: Naloxone prevents or blocks the action of narcotics (opioid medication). Naloxone is indicated for opioid overdose. Naloxone is sometimes prescribed to verify whether the client has overdosed with an opioid. 4. Incorrect: Clonidine is used to suppress opiate withdrawal symptoms. Serves effectively as a bridge to enable the client to stay opiate-free long enough to facilitate termination of methadone maintenance.

The nurse in the outpatient clinic performs an assessment on a client who takes propranolol for management of palpitations associated with mitral valve prolapse. Which statement by the client should be reported immediately to the primary healthcare provider? You answered this question Correctly 1. "My resting pulse was 60 this morning." 2. "I feel a little short of breath when walking." 3. "I have lost 5 pounds in the last 2 weeks." 4. "My blood pressure (BP) was lower this visit than last time."

RationaleStrategies 2. Correct: Propranolol is a non-selective beta blocker so it blocks sites in the heart and in the lungs. The shortness of breath could be the result of the adverse reactions of bronchospams or heart failure. This statement requires immediate investigation by the primary healthcare provider. 1. Incorrect: A side effect of propranolol is bradycardia. The client should be taught to contact their primary healthcare provider if their pulse is <50 beats per minute (bpm). A pulse rate of 60 bpm is acceptable. 3. Incorrect: Losing weight is not a side effect of propranolol. Weight loss regimen may be encouraged for hypertension. Losing 5 pounds in 2 weeks is within the acceptable range. 4. Incorrect: The therapeutic effect of propranolol is to reduce BP. If the client is asymptomatic, decreased BP is no big deal.

An elderly client is admitted to the floor with vomiting and diarrhea for three days. She is receiving IV fluids at 200 mL/hr via pump. What is the priority nursing action? You answered this question Incorrectly 1. Intake and output every shift. 2. Lung assessments every 2-4 hours. 3. Vital signs every shift. 4. IV site assessment every 2-4 hours.

RationaleStrategies 2. Correct: The IV is infusing at 200 mL/hr which is a rapid infusion rate for an elderly client. The lungs should be assessed every 2-4 hours to evaluate for potential fluid volume excess (FVE). 1. Incorrect: Input and Output (I&O) are important, but are a less priority than lung assessment in the elderly client. 3. Incorrect: Vital signs should probably be more frequent than every shift on the elderly client with dehydration. In addition, the cliet's IV rate is 200 mL/hr which may result in FVE. 4. Incorrect: The site should be monitored but will not be priority over lung assessment in the elderly client.

A client who has developed hypovolemic shock is receiving albumin. What assessment finding by the nurse indicates that the albumin has been effective? You answered this question Correctly 1. Swelling in the legs 2. Increase in uninary output 3. Proteinuria 4. Increase in waist measurement

RationaleStrategies 2. Correct: The action of albumin is to increase the serum albumin level. When the albumin level increases there is a shift of fluid from extracellular to intracellular. This action will result in an increase in urinary output. 1. Incorrect: This is a symptom of hypoalbuminemia. There is a shift in the fluid from intracellular to extracellular. This results in the swelling of the legs. 3. Incorrect: Hypoalbuminemia may cause damage to the kidneys. Proteinuria is indicative of renal disease or damage. 4. Incorrect: There may be a increased accumulation of fluid in the abdomen. The ascites is due to the decreased albumin level in the vascular space, which also causes damage to the liver.

The nurse is working at the triage station. Which client should the nurse triage first? You answered this question Correctly 1. A client with hepatitis A who states, "My arms and legs are itching." 2. A client with a cast on the right leg who states, "My right leg is killing me and nothing I do makes it stop hurting. " 3. A client with nausea and vomiting for two days states, "I am very weak and can't eat." 4. A client with hematuria and reports left flank pain.

RationaleStrategies 2. Correct: The client who has a cast with unrelieved severe pain indicates compartment syndrome and requires immediate action. This client is at greatest risk for harm because untreated compartment syndrome can cause irreparable nerve, and muscle damage and can lead to amputation. 1. Incorrect: A client with hepatitis A is not the highest priority at this time. The itching is most likely caused by accumulation of bile salts under the skin. The client will need to be evaluated but is not the triage nurse's highest priority. 3. Incorrect: This client would need to be seen and evaluated for dehydration. Of the clients listed here, this is not the triage nurse's highest priority. 4. Incorrect: When you see hematuria it leads you to worry about bleeding. Remember that even though hematuria may seem that the client is losing a lot of blood, it is not a significant loss. Hematuria is probably due to a kidney stone, infection, trauma or even prostate problems. In this triage scenario it is the lowest priority.

A client with a history of increasing dyspnea over the past week comes to the emergency department. After arterial blood gases (ABGs) are drawn, which information would be important for the nurse to document? You answered this question Correctly 1. The client had not been NPO prior to the test. 2. The client was on 2 L of oxygen by nasal canula. 3. Lung sounds are wet. 4. Client is sitting in upright position.

RationaleStrategies 2. Correct: The fact that the client is on 2 L/min of oxygen will affect the analysis of the ABG results. If the client is on oxygen, the partial pressure of oxygen (PO2) will be elevated due to the increased inhaled oxygen. 1. Incorrect: Whether the client has eaten or been NPO will not affect the evaluation of the ABG results. 3. Incorrect: An assessment of the client's lung sounds must be performed for a client with a history of dyspnea. This assessment will not directly affect the ABGs result. 4. Incorrect: The client's position will not directly affect the evaluation of the ABG results.

A client has been admitted with a diagnosis of septic shock and has been successfully intubated. The nurse performs and documents a rapid assessment. Which information from the assessment requires the most immediate action by the nurse? exhibit: Blood pressure 90/40 mm Hg Heart rate 112 beats/min Respiratory rate 32 breaths/min Temperature 103deg;F (39.4deg;C) axillary O2 saturation 94% 1. Lung assessment finding. 2. Blood pressure reading. 3. Elevated temperature 4. Urine description and output.

RationaleStrategies 2. Correct: The low blood pressure indicates that systemic tissue perfusion will not be adequate. The blood pressure needs to be improved rapidly. 1. Incorrect: The oxygen sat is 94%, so the adventitious lung sounds do not need immediate intervention. 3. Incorrect: The second priority is to treat the infection that is a likely cause of the temperature elevation and hypotension. 4. Incorrect: This is the likely cause of the sepsis, but the priority is to improve the BP. The second priority is to treat the infection.

An elderly Asian woman has been in the hospital for three weeks, and it seems that her condition is such that nursing home placement is in the client's best interest. The family is against placing their relative in the nursing home. How should the nurse respond to this? You answered this question Incorrectly 1. Encourage the family to accept nursing home placement as the best option for their loved one. 2. Listen to the family's concerns and report those to the primary healthcare provider. 3. Ask the client what she wants and tell the family to abide by the client's wishes. 4. Realize that the nurse does not need to be involved in this decision.

RationaleStrategies 2. Correct: The nurse should listen to the concerns of the family. The Asian culture tends to be opposed to nursing home placement and see it as their duty to care for their elders in the home. The nurse should listen and serve as an advocate. 1. Incorrect: The nurse should not impose personal values on the client and family. Cultures vary as to acceptance of nursing home placement. 3. Incorrect: The client may not be in a position to make this decision. 4. Incorrect: The nurse must serve as client advocate and intermediary between client/family and primary healthcare provider as decisions are made about this important issue.

A client has been prescribed chlorpromazine for the treatment of schizophrenia. The nurse makes afternoon rounds and finds the client's temperature to be 104.7º F/40.4º C. The client has extreme muscle rigidity, and the vital signs have been fluctuating for the last four hours. What should the nurse do first? You answered this question Incorrectly 1. Provide a tepid sponge bath. 2. Notify the primary healthcare provider immediately. 3. Administer an antipyretic immediately. 4. Administer the chlorpromazine as prescribed.

RationaleStrategies 2. Correct: These symptoms are consistent with neuroleptic malignant syndrome (NMS), which is an adverse reaction to antipsychotic drugs. The symptoms of NMS are fever, altered mental state, muscle rigidity, and autonomic dysfunction. This is a medical emergency, and immediate action should be taken. 1. Incorrect: The symptoms indicate a medical emergency and the need for an immediate response. The nurse should notify the primary healthcare provider first. 3. Incorrect: The high temperature should be assessed, but the extreme muscle rigidity and fluctuating vitals are a medical emergency. The client needs further immediate attention. 4. Incorrect: The nurse should not administer another dose of the antipsychotic medication due to the client's presenting symptoms. Usually, the primary healthcare provider would discontinue the medication immediately.

The occupational health nurse is leading a group discussion about addiction. What should the nurse include as the primary barrier to the client with alcohol addiction seeking treatment? You answered this question Correctly 1. Co-dependency 2. Denial 3. Depression 4. Stigma

RationaleStrategies 2. Correct: They reject that they have a drinking problem and will argue with you if you suggest it. The client with an addiction may also use denial to lessen the impact of their addiction. 1. Incorrect: Co-dependency makes alcohol abuse last longer, but this is not the reason they do not seek treatment. Persons with co-dependency have difficulty establishing healthy relationships. 3. Incorrect: No depression associated with substance abuse. The primary reason a person does not seek treatment is denial of their addiction. 4. Incorrect: Yes, clients may be afraid of the stigma associated with addiction recovery. The person must first address their denial of addiction issues.

A client is in the surgical suite to have a left total knee replacement. Prior to the surgeon initiating the first incision, what should the circulating nurse remind the surgical team to perform? You answered this question Correctly 1. Surgical scrub 2. Time-out 3. Sponge and instrument count 4. Inspection of the surgical site

RationaleStrategies 2. Correct: Time-out, done immediately before the procedure, is a final verbal verification of the correct client, procedure, site, and implant. Time-out is active communication among all members of the surgical/procedural team, initiated by a member of the team before surgery. 1. Incorrect: Surgical scrub should be done before entering the surgical suite. 3. Incorrect: The scrub nurse does instrument and sponge counts numerous times before, during, and after the procedure. 4. Incorrect: Simple inspection of the surgical site is not enough. The team must verbally communicate what is to be done, on what limb. Additionally, the limb should be marked as "This knee" or "yes".

Post thyroidectomy, the nurse assesses the client for complications by performing which assessment? You answered this question Correctly 1. Perform blood glucose monitoring every 6 hours 2. Check for a positive Chvostek's 3. Assess swallowing reflex 4. Monitor neck dressings for change in fit and comfort 5. Administer desmopressin per nasal spray for urinary output (UOP) greater than 200 mL/hr

RationaleStrategies 2., 3., & 4. Correct: A positive Chvostek's and Trousseau's is indicative of tetany (low calcium). This can occur when one or more of the parathyroids are accidently removed when the thyroid is removed. A weak, raspy voice, swallowing difficulty, and impaired respiratory status can be caused by nerve injury. Change in fit and comfort of the dressing can indicate possible neck swelling, which can affect the airway. 1. Incorrect: A possible complication of a thyroidectomy is to remove one or more parathyroid glands. The parathyroids' action is to regulate the serum calcium levels. The parathyroid does not regulate the blood glucose levels. 5. Incorrect: The action of desmopressin is to increase the reabsorption of water in the kidney. A decrease in vasopressin, (antidiuretic hormone) is not a complication of a thyroidectomy.

The nurse is teaching a client regarding buspirone. The nurse recognizes that teaching has been effective when the client makes which statements? You answered this question Correctly 1. "I should start feeling better in two or three days." 2. "I should not drink alcohol while taking this medication." 3. "I will rise slowly from lying to sitting or standing." 4. "I will notify my primary healthcare provider of any unusal facial movements." 5. "I need to keep the medication in a closed container in the refrigerator."

RationaleStrategies 2., 3., & 4. Correct: These are correct statements that indicate that the client understands the teaching about this medication. Combined with alcohol use, the client may develop dizziness or drowsiness. Buspirone may cause orthostatic hypotension, so position changes are made slowly. The primary healthcare provider should be notified of any abnormal facial movements. The therapeutic effect occurs in 3-4 weeks. 1. Incorrect: Buspirone has a lag time of 10-14 days between start of therapy and subsiding of symptoms. The client must take the medication regularly, as ordered, so that it has sufficient time to take effect. 5. Incorrect: Store this medication in a container at room temperature below 86°F (30°C). The container should be kept away from light and moisture.

A client has been admitted to the medical unit and placed on airborne precautions for suspected pulmonary tuberculosis (TB). The nurse will assess for which signs and symptoms? You answered this question Incorrectly 1. Weight gain 2. Fatigue 3. Hemoptysis 4. Diaphoresis during sleep 5. Anorexia

RationaleStrategies 2., 3., 4. & 5. Correct: Feeling tired all the time, loss of appetite, fever, coughing up blood, and night sweats are the most common signs and symptoms of active TB. 1. Incorrect: A symptom of TB is a decreased desire for food. This will result in weight loss rather than weight gain.

A nurse is calling the primary healthcare provider about a client who is experiencing dyspnea and chest pain two days post total knee replacement. Which statements by the nurse are appropriate according to the communication tool SBAR (Situation, Background, Assessment and Recommendation)? You answered this question Correctly 1. "Hello Dr, I am calling about one of your clients." 2. "Jane Doe is having increasing dyspnea and is reporting chest pain." 3. "Jane Doe had a total knee replacement two days ago. Pulse is 120, BP 128/54, Resp 32. She is restless." 4. "From my assessment, I think she may be having a cardiac event or a pulmonary embolism." 5. "I recommend that you see the client immediately and that we start oxygen stat. Do you agree?"

RationaleStrategies 2., 3., 4., & 5. Correct: First, the nurse should identify self, agency, and client calling about. Then deliver SBAR. The Situation, Background, Assessment and Recommendation (SBAR) technique has become the Joint Commission's stated industry best practice for standardized communication in healthcare, effortlessly structuring critical information primarily for spoken delivery. Each of these statements fulfills appropriate SBAR requirements. 1. Incorrect. The nurse should identify the primary healthcare provider by name and should then identify self, the agency the nurse is calling from and the client by name. For instance: "Dr. Smith, this is nurse Adams, RN. I am calling about your client, Jane Doe, at ABC hospital."

Which nursing tasks can the RN delegate to an unlicensed assistive personnel (UAP)? You answered this question Correctly 1. Tell a female client who has recurrent urinary tract infections how to wipe after urinating. 2. Obtain blood pressure of client diagnosed with nephrotic syndrome. 3. Collects a urine specimen from an indwelling catheter tubing. 4. Document the intake and output of a client in acute renal failure. 5. Irrigate the foley catheter of a client who has had transurethral resection of the prostate (TURP). 6. Perform perineal care of a client who has urinary incontinence.

RationaleStrategies 2., 4., & 6. Correct: These are all tasks that can be performed by the UAP. The UAP has received training for completing these tasks. 1. Incorrect: The UAP cannot provide teaching; that is planned and implemented by the RN. 3. Incorrect: This is out of the scope of practice for the UAP as it is requires entering a sterile system using sterile technique. 5. Incorrect: The UAP does not have the knowledge and skill to irrigate catheters of any kind. This is a skilled procedure.

Which findings would indicate to the nurse that a client with Addison's disease has received too much glucocorticoid replacement? You answered this question Incorrectly 1. Dry skin and hair 2. Hypotension 3. Rapid weight gain 4. Decreased blood glucose level 5. Increased cholesterol

RationaleStrategies 3, & 5. Correct: Excessive drug therapy with glucocorticoids will cause rapid weight gain, round face, and fluid retention. Cholesterol and triglycerides in the blood are also increased by glucocorticoids. Long term use of high steroid doses can lead to symptoms such as thinning skin, easy bruising, changes in the shape or location of body fat (especially in your face, neck, back, and waist), increased acne or facial hair, menstrual problems, impotence, or loss of interest in sex. 1. Incorrect: Dry skin and hair would be seen with a decrease in sex hormones, not with a increase in glucocorticoids. An increase in glucocorticoids will result in an increase in oil production in the skin. 2. Incorrect: Hypotension is a sign of Addison's disease. The client loses sodium and water, causing the client's blood pressure to drop. This loss of sodium and water would come from a decrease in mineralocorticoids. This would have nothing to do with glucocorticoids. 4. Incorrect: An increase in glucocorticoids will result in glucose intolerance. The client will become resistive to insulin production. This will result in an increase in the serum blood glucose.

The primary healthcare provider suspects the client has tuberculosis (TB) and prescribes a Mantoux test. What precautions should the nurse take when administering the Mantoux test? You answered this question Correctly 1. Don sterile gloves. 2. Place the client on reverse isolation. 3. Wear a particulate respirator 4. Obtain a consent form. 5. Initiate airborne precautions.

RationaleStrategies 3. & 5. Correct: A disposable particulate respirator that fits snugly around the face is needed. The client needs to be on acid-fast bacilli (AFB) isolation precautions, not reverse isolation. Airborne precautions include a private room with negative pressure and a minimum of 6 air exchanges per hour. Ultraviolet lamps and high efficiency particulate air filters are also needed. 1. Incorrect: Sterile gloves are not needed. Standard precautions indicate clean gloves. 2. Incorrect: The client needs to be on airborne isolation precautions, not reverse isolation. Airborne precautions include a private room with negative pressure and a minimum of 6 air exchanges per hour. Ultraviolet lamps and high efficiency particulate air filters are also needed. 4. Incorrect: A consent is not necessary. SIC-117

The nurse is caring for a client with pneumococcal pneumonia. Which nursing observations would indicate a therapeutic response to the treatment regime for the infection? You answered this question Correctly 1. Dyspnea on exersion with nonproductive cough 2. Tachypnea with use of accessory muscles 3. Expectorating moderate amounts of thin, white sputum 4. White blood cell count of 18,000 cells per mcL 5. Crackles clearing with cough

RationaleStrategies 3. & 5. Correct: The client has no signs of active infection. A cough with thin, white sputum is expected for a while, but it is infection free. Crackles clearing with cough are signs of an effective cough effort. 1. Incorrect: The client still has signs of active infection and complaints of dyspnea. The client should not have a nonproductive cough on exertion. 2. Incorrect: The client still has signs of active infection, such as tachypnea with use of accessory muscles. The respiration rate for an adult is 12-20 per minute with no use of accessory muscles. 4. Incorrect: The white cell count is still too high. A normal range for white blood count is between 4,500 and 10,000 mcL. This is not a therapeutic response to the treatment.

During evening rounds, the nurse discovers that a violent client with a history of threats against a former girlfriend cannot be located. The client's window is open and personal belongings missing. Based on recent threats of violence against the girlfriend, what is the nurse's initial action? You answered this question Incorrectly 1. Look for the client quietly to maintain confidentiality. 2. Notify the local police to organize a search party. 3. Initiate the missing client protocol. 4. Complete an "Against Medical Advice" (AMA) form on the client's elopement.

RationaleStrategies 3. Correct. Since the client is missing and is considered to be a risk to himself or others, the missing client protocol is immediately implemented. A organized and escalating search for the client will occur. 1. Incorrect. Although HIPAA is very precise on the issue of confidentiality, privacy is waived in specific, extreme situations that involve the life of a client, staff, or family. While the nurse may alert staff members to help search the building or facility grounds, this is not the initial action of the nurse. 2. Incorrect. Although local police may eventually be asked to assist in locating the client, this decision will be made by the facility's administrative personnel. However, this is not the initial action under the stated circumstances. 4. Incorrect. An Against Medical Advice (AMA) form is signed by a client who decides to leave a facility without treatment, against the advice of the primary healthcare provider. That form does not apply in this situation since the client has already left. This event falls under the category of elopement.

A client arrives in the emergency department with fever, nuchal rigidity, and seizures. What action should the nurse take first? You answered this question Correctly 1. Administer Penicillin IVPB. 2. Obtain blood cultures from two sites. 3. Place on droplet precautions. 4. Set up for lumbar puncture.

RationaleStrategies 3. Correct. When bacterial meningitis is suspected, the nurse should place the client on droplet precautions at once. Transmission can occur by the droplet/close contact route for up to 24 hours even after starting effective antibiotic therapy. The Centers for Disease Control and Prevention (CDC) recommends droplet precautions in addition to Standard Precautions for bacterial meningitis. 1. Incorrect. Penicillin is the drug of choice but would be initiated after blood cultures and other cultures are obtained. The client would be placed in isolation prior to starting penicillin. 2. Incorrect. Lumbar puncture is done to obtain cultures for diagnosis but would be done after placing in isolation. Blood cultures are obtained from the lumbar puncture and cultures may be obtained from blood, nasopharynx, urine, or skin lesions. The client would be placed in isolation first. 4. Incorrect. A lumbar puncture and cerebrospinal examination are needed to confirm a diagnosis. The lumbar puncture should be performed after placing the client in droplet precautions.

A child is being admitted to the hospital with a diagnosis of acute glomerulonephritis. In performing the history and physical, what would be a priority assessment that the nurse should include when questioning the child and caregivers? You answered this question Correctly 1. Types of contact sports played 2. Amount of acetaminophen intake 3. Recent sore throat 4. Recent exposure to salmonella

RationaleStrategies 3. Correct: Acute post-streptococcal glomerulonephritis (APSGN) results from a group A beta-hemolytic streptococci infection that originates typically in the throat (strep throat) or the skin (impetigo). The strep bacterial infection can cause the filtering units of the kidneys (glomeruli) to become inflamed and results in a decreased ability of the kidneys to filter the urine. The disorder may develop 1-2 weeks after an untreated throat infection or 3-4 weeks after a skin infection. 1. Incorrect: Glomerulonephritis is not associated with trauma over the kidney region, so questioning about contact sports would not be relevant to glomerulonephritis. 2. Incorrect: Excessive acetaminophen intake can cause liver damage but is not associated with glomerulonephritis. 4. Incorrect: Salmonella is not an organism that is linked to the development of glomerulonephritis. Group A beta-hemolytic streptococci is the causative organism for acute glomerulonephritis.

Which factor would most likely predispose a client to developing shock following a fracture of the femur? You answered this question Incorrectly 1. Pooling of the blood in the lower leg 2. Generalized vasoconstriction in the lower extremities 3. Loss of blood into soft tissues surrounding the fracture 4. Depression of the adrenal gland by toxins released at the injury

RationaleStrategies 3. Correct: After a fracture, the factor that would most likely lead to shock is loss of blood into the soft tissue surrounding the fracture. When fractures occur, the ends of the bones can sever major arteries, causing loss of blood into the surrounding tissue. 1. Incorrect: If the leg with the fracture is placed in a dependent position for an extended time, bleeding into the surrounding tissue is more acute. Pooling of blood in the lower extremities is a possibility. 2. Incorrect: Vasoconstriction in the lower extremities is an appropriate compensatory response for a "shocky" client. Vasoconstriction of the lower extremities will increase blood pressure. 4. Incorrect: The adrenal glands are located on top of the kidneys and control steroids. They are not depressed by toxins related to injury, but it sounded good, right?

What is priority for the nurse to determine about a client who is scheduled for a tubal ligation in the outpatient surgical center? You answered this question Incorrectly 1. Client's prior experiences with outpatient surgery. 2. Medical plan and the extent of insurance coverage for outpatient surgery. 3. Client's plan for transportation and care at home. 4. Client's plan to spend the night at the surgical center.

RationaleStrategies 3. Correct: After outpatient surgery, the client should not be allowed to drive home. A driver and assistance at home are necessary prior to discharge. 1. Incorrect: The client's prior experience would be a factor in the pre-operative phase. 2. Incorrect: The medical plan's coverage would not be assessed by the business office in the planning phase. This is done several days or weeks prior to the scheduled surgery. 4. Incorrect: It would be atypical for the client to spend the night in a surgical clinic, as they are not generally open at night for overnight medical stay.

A client recently diagnosed with diabetes is sent home with a prescription for subcutaneous insulin. What statement made by the client indicates that teaching has been effective regarding safe needle disposal? You answered this question Correctly 1. "I should wrap the needle in a paper towel and place in the trash." 2. "I should use a hospital issued biohazard container for all needles." 3. "I may use any hard plastic container with a screw-on cap." 4. "I should take my needles to the nearest hospital for disposal. "

RationaleStrategies 3. Correct: At home, an FDA approved sharps container is not needed, however, needles, syringes, and sharps may be disposed of in a hard plastic container. Clients should follow their community guidelines for sharps container disposal. This protects the sanitation engineers from injury by the sharps. 1. Incorrect: Syringes must be placed in a safe container in order to protect others from becoming injured by sharps. Wrapping the needle in a paper towel and placing in the trash increases the possibility of injury to someone. 2. Incorrect: The hospital is not involved in sharps disposal in the home. A hard plastic container with a screw on cap is an acceptable container to dispose of needles. 4. Incorrect: The hospital is not involved in sharps disposal in the home. The client can dispose of needles safely at home in a hard plastic container with a screw on cap. The needle should not be brought to the hospital for disposal.

A client who has been on a psychiatric unit because of several attempted suicides states, "I am happy to be going home today." What is the nurse's best analysis of this statement? You answered this question Correctly 1. No longer has depression. 2. Has developed appropriate coping mechanisms. 3. May have decided on another suicide plan. 4. Is happy to go home and see family.

RationaleStrategies 3. Correct: Clients who have attempted suicide may come up with another plan. Once they do, they are generally happy, satisfied, and begin giving away personal belongings. Clients usually exhibit some trepidation about leaving the hospital. 1. Incorrect: This cannot be determined by the client's statement. Suicidal depression is not a short term problem, but requires intense therapy. 2. Incorrect: Always assume the worse until proven otherwise. The client's statement does not reflect an increase in their coping mechanism. The suicide client may have planned another suicide attempt. 4. Incorrect: This cannot be determined by the client's statement. The client said nothing about family. There is no data to identify the client's family relationships.

The nurse is caring for a newly diagnosed diabetic in diabetic hyperosmolar hyperglycemic nonketotic (HHNK) state. What does the nurse anticipate the immediate treatment plan for this client will include? You answered this question Incorrectly 1. NPH insulin. 2. Potassium 40 mEq (40 mmol/L) slow intravenous push. 3. Intravenous administration of isotonic saline. 4. Intravenous sodium bicarbonate.

RationaleStrategies 3. Correct: Clients who present with HHNK are severely dehydrated. This is because high glucose levels in the vascular space lead to particle induced diuresis (PID) or osmotic diuresis. The clients lose large amounts of volume out of vascular space. The client may even be "shocky". Isotonic saline is the treatment of choice for clients who are in HHNK. Isotonic saline will go into the vascular space and stay there thus improving the fluid volume deficit that has developed. 1. Incorrect: In HHNK, long acting insulin such as NPH or glargine would be inappropriate for the emergent treatment of HHNK. The blood sugar needs to be controlled quickly with a short acting insulin. Regular insulin needs to be given IV. 2. Incorrect: First, never give potassium IVP! Hypokalemia or hyperkalemia may be present in HHNK. However, it is generally not the initial treatment until after the client starts receiving insulin, which moves potassium out of the blood and into the cell. Serum potassium may be elevated due to an extracellular shift caused by insulin deficiency. However, total body potassium is likely low regardless of its serum value; a low measured serum potassium suggests profound total body losses, and clients should be placed on cardiac monitoring. During treatment, insulin drives potassium into cells, and intravenous (IV) hydration dilutes potassium in the circulation. Aggressively replace potassium to maintain plasma levels in the normal range during treatment. 4. Incorrect: IV bicarbonate is not indicated for initial treatment of HHNK. The body is not burning fat for energy, there are no ketones. No ketones, no acidosis.

A client calls the clinic to ask the nurse if it would be okay to take the herbal medication kava-kava to help reduce anxiety. What is the nurse's best response? You answered this question Correctly 1. "Why do you want to take kava-kava?" 2. "I really doubt your primary healthcare provider will approve you taking kava-kava." 3. "Kava-kava can cause liver damage, so we need to consult your healthcare provider." 4. "Do not take Kava-kava for more than a year without a primary healthcare provider's supervision."

RationaleStrategies 3. Correct: Kava-kava can cause liver damage. It is recommended that if if taking kava-kava the client should be under the direct supervision of a primary healthcare provider. 1. Incorrect: The client has already answered that: anxiety. This question will put the client on the defensive. This is an example of nontherapeutic communication technique asking for explanations. 2. Incorrect: Judgmental response. This will put the client on the defensive. This is an example of the nontherapeutic communication technique of an aggressive response. 4. Incorrect: You should not take this drug for longer than 3 months without a primary healthcare provider's supervision. There have been recent reports that kava-kava causes liver damage.

A client at 36 weeks gestation is receiving magnesium sulfate for treatment of pre-eclampsia. Which finding by the nurse requires immediate action? You answered this question Incorrectly 1. Respiratory rate of 12 2. Deep tendon reflexes (DTR) of 3+ 3. Urinary output (UOP) of 100cc/4hours 4. Fetal heart rate (FHR) of 120

RationaleStrategies 3. Correct: Magnesium sulfate is a potent central nervous system depressant that is excreted through the kidneys. Adequate kidney function is vital to prevent magnesium toxicity. A urinary output of at least 30 mL/hr is the minimum standard to evaluate adequate kidney function. 1. Incorrect: A respiratory rate of 12 is within the acceptable range for the client. Magnesium sulfate can cause bradycardia, tachycardia, or irregular rhythm. 2. Incorrect: Magnesium sulfate causes decreased DTRs. A 3+ DTR is a very brisk response and does not reflect a symptom of magnesium sulfate toxicity. 4. Incorrect: Fetal heart rate of 120 is within the normal range of 110-160 bpm. A heart rate of 110-120 tells the nurse to be "worried and watching", but the range is acceptable.

The nurse is providing care to a client who has a large abdominal dressing. Which intervention is most likely to reduce the risk of skin irritation due to frequent dressing changes? You answered this question Correctly 1. Use a paper tape for adhering the dressing. 2. Use tape sparingly. 3. Secure the dressing with Montgomery straps. 4. Change the dressing only if it becomes saturated with drainage.

RationaleStrategies 3. Correct: Montgomery straps will allow the dressing to be held in place without the use of tape. The adhesive on the ends of the straps is the only adhesive used. 1. Incorrect: Paper tape may be less irritating; however, with repeated changes, skin irritation is more likely. Montgomery straps will decrease the repeated tape changes. 2. Incorrect: Use of the tape should be sufficient to secure the dressing and applied in a way to allow mobility if placed over a joint. There still is an increase of skin irritation from applying the tape. 4. Incorrect: The soiled dressing is a medium for bacteria growth. The dressing should be changed as ordered or required.

The nurse is preparing to provide oral care to an unconscious client. What is the most important step for the nurse to provide? You answered this question Correctly 1. Performing hand hygiene. 2. Explaining the procedure to the family. 3. Positioning the client in side-lying position. 4. Raising the head of bed 30 degrees.

RationaleStrategies 3. Correct: Positioning the client in a side-lying position allows secretions to drain from the mouth and prevents aspiration. The most important aspect of care is the protection of the airway of this unconscious client. This is accomplished through proper positioning of the client in a side-lying position. 1. Incorrect: Hand hygiene is a key component of standard precautions. Hand hygiene is not a priority over preventing aspiration. 2. Incorrect: Informing the family about the procedure should be done, but is not the most important step in oral care. The nurse should explain the oral care procedure to the family. Maintaining the clients airway is the priority action. The client should be placed in the side lying position. 4. Incorrect: No, side lying is the appropriate position to allow drainage of secretions from mouth and prevent aspiration. Positioning the client with the HOB elevated to 30 degrees will not promote drainage of secretions from the client's mouth. The priority action is to maintain the client's airway.

A client was prescribed thioridazine hcl five days ago and presents to the emergency department with a shuffling gait, tremors of the fingers, drooling, and muscle rigidity. Which adverse reaction to this medication does the nurse suspect? You answered this question Correctly 1. Akinesia 2. Neuroleptic malignant syndrome 3. Pseudoparkinsonism 4. Oculogyric crisis

RationaleStrategies 3. Correct: Pseudoparkinsonism may appear 1 to 5 days following initiation of antipsychotic medications: occurs most often in women, the elderly, and dehydrated clients. Symptoms include tremor, shuffling gait, drooling, and rigidity. 1. Incorrect: Akinesia is defined as muscle weakness. This client is not presenting with this symptom. 2. Incorrect: Neuroleptic malignant syndrome is a rare, but fatal complication of neuroleptic drugs. Symptoms include hyperpyrexia up to 107 degrees, tachycardia, tachypnea, fluctuations in BP, diaphoresis, coma. 4. Incorrect: Oculogyric crisis is uncontrolled rolling back of the eyes and may appear as part of dystonia (involuntary muscular movements of face, arms, legs, and neck). Oculogyric crisis is not a side effect of thioridazine.

Which client requires immediate intervention by the nurse? You answered this question Correctly 1. Client diagnosed with Crohn's disease reporting frequent bloody diarrhea and abdominal cramping. 2. Client with renal calculi who reports no pain relief from ketorolac administered 30 minutes ago. 3. Client with a fractured femur reporting sharp chest pain of 4/10. 4. Client admitted with cholelithiasis reporting right-sided abdominal pain of 8/10.

RationaleStrategies 3. Correct: Sharp chest pain after a fractured femur could indicate a pulmonary embolus (PE) or a fat embolus and requires immediate intervention by the nurse. 1. Incorrect: This is not the most life-threatening problem. The client with suspected PE or fat embolus takes priority, although this client would be closely monitored for fluid and electrolyte imbalances. 2. Incorrect: This is not the most life-threatening problem. The client with suspected PE or fat embolus takes priority. Just remember, pain never killed anybody. 4. Incorrect: This is not the most life-threatening problem. The client with suspected PE or fat embolus takes priority. Although this client's pain does not need to be ignored, it doesn't take priority over someone with an embolus.

What should a nurse teach a client who has been diagnosed with hepatitis A? You answered this question Incorrectly 1. Hepatitis A is spread through blood and body fluid. 2. Chronic liver disease is a common complication of hepatitis A. 3. Symptoms of hepatitis A include malaise, dark colored urine, and jaundice. 4. Treatment includes alpha-interferon and ribavirin.

RationaleStrategies 3. Correct: Symptoms of hepatitis A include fever, malaise, loss of appetite, diarrhea, nausea, abdominal discomfort, dark-colored urine, and jaundice. 1. Incorrect: Hepatitis A is spread when an uninfected person ingests food or water that is contaminated with the feces of an infected person. This disease is closely associated with unsafe water, inadequate sanitation, and poor personal hygiene. 2. Incorrect: Hepatitis A infection does not cause chronic liver disease and is rarely fatal, but it can cause debilitating symptoms and fulminant hepatitis, which is associated with high mortality. 4. Incorrect: Used for hepatitis C.

The primary healthcare provider prescribes a combination of pyrazinamide and isoniazid to treat a client with tuberculosis. The client asks the nurse, "Why am I taking two drugs?" Which explanation should the nurse give the client? You answered this question Correctly 1. One diminishes the side effects of the other. 2. Hepatoxicity is reduced. 3. Bacterial resistance is decreased. 4. One kills the live bacteria, and the other the spores.

RationaleStrategies 3. Correct: The CDC says that the initial phase of treatment for newly diagnosed cases of pulmonary TB should consist of a multiple-medication regimen because many cases of TB are caused by strains of the bacteria that are resistant to isoniazid or rifampin. This client has been prescribed the multiple medication regimen of pyrazinamide and isoniazid. 1. Incorrect: These drugs do not diminish the side effects of each other. In fact, they potentiate them. 2. Incorrect: Both drugs can cause hepatoxicity. 4. Incorrect: Isoniazid is bacteriostatic or bactericidal against susceptible bacteria. Pyrazinamide is also bacteriostatic against susceptible bacteria.

Which pediatric client should the nurse see first? You answered this question Incorrectly 1. Six year old with a femur fracture. 2. Two year old with a fever of 102 ° F (38.8 ° C) 3. Three year old with wheezes in right lower lobe. 4. Two year old whose gastrostomy tube came out.

RationaleStrategies 3. Correct: The child having respiratory difficulty should be seen first. This is an example of using Maslow to set priorities. Airway will always be first followed by breathing and circulation. This client is not stable. 1. Incorrect: This client will need to be seen, but not prior to a client with an immediate vital function problem such as airway and breathing. From the information given, all we know is that the child has a fracture so we have to assume the client is stable. 2. Incorrect: This client will need to be assessed, but not prior to a client with an immediate vital function problem such as airway and breathing. The temperature is elevated but there is no information to cause the nurse to think the situation is life threatening. 4. Incorrect: The tube has come out and needs to be replaced so that feeding can be resumed. You have time before this client is in any distressed so this is not your priority client.

The nurse is performing the admission assessment on a client who is having a breast augmentation. Which client information would be most important for the nurse to report to the surgeon before surgery? 1. Client is concerned about who will care for her two children while she recovers. 2. There is a history of postoperative dehiscence after a previous C-section. 3. Client's last menstrual period was 8 weeks ago. 4. Client is concerned over pain control postoperatively.

RationaleStrategies 3. Correct: The client may be pregnant, so a pregnancy test will need to be completed prior to administering anesthetic agents. As you look at these options they are all possible but only one is a priority and in this case life threatening. 1. Incorrect: Adequate caregivers can be discussed with the client without contacting the primary healthcare provider. This is important but not the priority to report to the surgeon. 2. Incorrect: Every person who has a surgical wound is at risk for dehiscence especially in the first two weeks after surgery. Educate the client concerning signs and symptoms and causes of dehiscence but this is not your priority here. 4. Incorrect: The client's postoperative pain control will be discussed both before and post surgery. Always discuss clients concerns prior to surgery and consult the primary healthcare provider if you are unable to satisfy the client.

The nurse is teaching a class to primiparas on breastfeeding. How many extra kilocalories per day would the nurse instruct the class participants to consume post-delivery to compensate for the increased energy requirements of lactation? You answered this question Correctly 1. 1000 2. 300 3. 500 4. 800

RationaleStrategies 3. Correct: The client needs an extra 500 kcal/day above the usual allowance because the average woman will secrete between 425-700 kcals per day in her breast milk. By increasing the daily caloric intake by 500 kcal the client will offset these losses. 1. Incorrect: 1000 kcal/day is more calories than are needed to offset the caloric loss of breastfeeding. 2. Incorrect: 300 kcal/day is not enough calories to offset the caloric loss of breastfeeding. 4. Incorrect: 800 kcal/day is more calories than are needed to offset the caloric loss of breastfeeding.

The charge nurse has received report from the emergency department about a client diagnosed with Cushing's disease being admitted to the unit. Which client in a semi-private room would be appropriate for the charge nurse to have this client share? You answered this question Correctly 1. Client who has leukemia. 2. Client diagnosed with gastroenteritis. 3. Client who has a fractured hip. 4. Client diagnosed with bronchitis.

RationaleStrategies 3. Correct: The client with Cushing's disease could go in the room with the client who has a fractured hip, as this client does not have an infection. 1. Incorrect: Both of these clients are immunocompromised and should not share a room with each other. 2. Incorrect: The client with gastroenteritis poses a risk of infection to the client with Cushing's disease because this client is immunosuppressed. 4. Incorrect: The client with bronchitis poses a risk of infection to the client with Cushing's disease.

A client with dementia has been admitted to the medical floor. The family informs the nurse that the client tends to wander at night. When planning client safety goals, which action by the nurse would take priority? You answered this question Correctly 1. Place client with a roommate who is able to notify staff when client wanders. 2. Discuss safety goals with family, encouraging them to spend time with client. 3. Designate an unlicensed assistive personnel (UAP) to sit with the client through the night. 4. Reorient the client every shift regarding floor policies and safety procedures.

RationaleStrategies 3. Correct: The family specifically informed the nurse that the client wanders at night. When preparing care plan safety goals, the nurse understands the priority is to ensure client safety and have staff personnel to stay with the client during those hours of wandering. This action does not require a licensed nurse; therefore, a unlicensed assistive personnel UAP is the most appropriate staff to sit with the client. 1. Incorrect: It is neither ethical nor legal to expect another client to be accountable for a roommate's behavior or whereabouts. 2. Incorrect: The nurse would indeed discuss plans for client safety with the family and encouraging the family to spend time with the client can be helpful. However, this does not release the staff from the responsibility for the client's safety. 4. Incorrect: This client is known to have dementia and re-explaining policies and safety procedures every shift would not ensure this client's compliance, understanding or safety.

A teenage client is placed on life-support as a result of a motor vehicle accident (MVA). Following an electroencephalogram (EEG), the client has been declared brain dead. Which action by the nurse would take priority? You answered this question Correctly 1. Call the respiratory department to have the ventilator removed. 2. Notify the facility's pastoral personnel. 3. Contact the regional organ procurement team. 4. Ask the family to select a funeral home.

RationaleStrategies 3. Correct: The first priority is to notify the designated organ procurement team. These personnel are trained to determine if the client would be an appropriate donor, how to approach the grieving family and discuss options, and to make any necessary arrangements in such a situation. Time is of the essence in the case of organ donation. Even if the family refuses to donate organs, it is the Procurement Team that will deal with the situation. 1. Incorrect: Although the Respiratory department may be present at the time of extubation, removing the ventilator from the room is not a priority at this time. The client will most likely remain on the ventilator until the Procurement Team has talked with the family. 2. Incorrect: The facility pastoral staff may already be aware of the critical nature of this situation, and the family may have also gathered their own support group at this time. Though the nurse may notify hospital's support personnel, this would not be the first priority action. 4. Incorrect: While the family will have to make many difficult decisions in this situation, the nurse must focus on the client first. Time is crucial for organ viability, and the Organ Procurement Team needs to mobilize appropriate personnel to deal with all issues.

The nurse is planning daily activities for a client who has a diagnosis of schizophrenia. The client tends to spend most of the time in bed and is very uncomfortable when other clients are in the day area of the unit. What activity would be most therapeutic for this client? You answered this question Correctly 1. Watching TV with two other clients in the day room. 2. Watching TV alone in a conference room. 3. Spending time in brief one on one interactions with the nurse. 4. Sitting in the day-room away from other clients.

RationaleStrategies 3. Correct: The interaction with the nurse can keep the conversation reality based and provide interaction with someone. Clients with schizophrenia may be very withdrawn and need the presence of the nurse. 1. Incorrect: The client is very uncomfortable around the other clients. This action could be appropriate as the client's condition begins to stabilize. 2. Incorrect: The client needs interaction with others. Time with others allows the client to stay reality based. When alone, there may be more time for delusional thought or auditory hallucinations. 4. Incorrect: The nurse can assess the thought processes of the client and offer acceptance of the client. Sitting away from other clients is not recommended therapeutic intervention for this client.

A child was diagnosed with attention-deficit/hyperactivity disorder (ADHD) in the clinic one week ago. Today the child's mother calls the clinic to tell the nurse, "My child has not been able to sleep since being put on methyphenidate." What is the best response for the nurse to make? You answered this question Correctly 1. "I will discuss this with the primary healthcare provider. A different medication may be prescribed." 2. "The insomnia will get better over time. Just wait it out." 3. "To prevent insomnia, give your child the last daily dose at least 6 hours before bedtime." 4. "Your child may have overdosed on the medication. Go to the emergency department now."

RationaleStrategies 3. Correct: The last dose should be at least 6 hours before bedtime. This will decrease the child's difficulty in falling asleep. If the medication is sustained-released, administer the dose in the morning. 1. Incorrect: This is premature. The nurse should identify the problem and suggest an alternative response. Try changing the time to help with sleep. 2. Incorrect: The child may adjust to the medication, but this will not correct the problem of insomnia. An alternative intervention should be recommended. 4. Incorrect: The client has not overdosed based on the data presented. Telling the mother to take the child to the emergency room is not needed.

A hospice nurse is assessing a client reporting chronic pain (5/10 on the pain scale). In addition to the primary healthcare provider and the nurse, what member of the care team will assist in providing comfort therapies for this client? You answered this question Incorrectly 1. Physical therapist 2. Nutritionist 3. Massage therapist 4. Occupational therapist

RationaleStrategies 3. Correct: The massage therapist provides alternative therapies that complement the medical pain control therapies being provided by the primary healthcare provider and the nurse. 1. Incorrect: A physical therapist is trained to improve and restore mobility. Due to the client's terminal status short term reduction of the pain is needed. 2. Incorrect: The nutritionist's main focus is on the client's nutritional status. This will not affect the pain level of the client. 4. Incorrect: The occupational therapist's main focus is to improve the client's ability to perform activities of daily living and work skills. Due to the terminal status of this client, pain management is the goal for this client.

A woman, diagnosed with an ectopic pregnancy, asks the nurse the purpose of receiving methotrexate. What is the best reply for the nurse to make? You answered this question Correctly 1. "Methotrexate will stop your bleeding." 2. "It will destroy fetal cells that got into your blood so that antibodies will not be formed." 3. "This medication will stop the growth of the embryo to save your fallopian tube." 4. "Cervical dilation is expected after receiving this medication."

RationaleStrategies 3. Correct: The medical management of an ectopic pregnancy is to prescribe methotrexate. The action of methotrexate is to stop the growth of the embryo in the fallopian tube. The embryo is reabsorbed and the fallopian tube can be saved. 1. Incorrect: Methotrexate does not stop bleeding. It will stop the growth of the embryo so that the fallopian tube can be saved. 2. Incorrect: RhoGam is given to destroy fetal cells that got into mom's blood so that antibodies are not formed. This is done when mom is Rh negative. 4. Incorrect: Methotrexate does not cause cervical dilation. This medication will prevent damage to the fallopian tube by halting the growth of the embryo.

When assessing a client's testes, which finding would indicate to the nurse the need for further investigation? You answered this question Correctly 1. Rope like area located at the top of the back of a testicle. 2. Right testicle is slightly larger than the left testicle. 3. Lump the size of a piece of rice. 4. Nonpalpable lymph nodes in groin.

RationaleStrategies 3. Correct: The most common symptoms of testicular cancer are a painless enlargement of one testis and the appearance of a palpable small hard lump on the front or side of the testicle.. 1. Incorrect: This is a normal finding. The epididymis feels soft, rope like, and is slightly tender to pressure, and is located at the top of the back part of each testicle. 2. Incorrect: It is normal for one testicle to be slightly larger than the other for most males. 4. Incorrect: Lymph node in the groin are not normally palpable.

The nurse is caring for a client with renal failure. The client has a 24 hour intake of 2500 mL and a 24 hour urinary output of 200 mL. What is the priority nursing assessment? You answered this question Correctly 1. Assess for dependent edema. 2. Monitor for cardiac arrhythmias. 3. Auscultate breath sounds. 4. Monitor sodium and potassium levels

RationaleStrategies 3. Correct: The nurse is "worried" about fluid volume excess. In fluid volume excess (FVE), the number one concern is heart failure with resultant pulmonary edema. In FVE, you can stress the heart so much that the heart begins to fail. With heart failure, the cardiac output decreases. With decreased cardiac output, there is decreased forward flow out of the heart. With decreased forward flow there is back flow. Back flow from the left ventricle results in fluid accumulation in the lungs. The best assessment for heart failure is to auscultate lung sounds. 1. Incorrect: Inspecting for dependent edema does not address the biggest problem/concern in FVE. The nurse is "worried" about pulmonary edema. This client will probably have edema, but it is not more important than breath sounds. 2. Incorrect: After evaluating the output versus the input amounts, the lungs should be assessed to evaluate the pressure of FVE. Cardiac arrhythmias are a possibility, due to the stress on the heart due to FVE. 4. Incorrect: Electrolytes may be abnormal due to FVE. The number one concern is FVE and pulmonary edema.

A newborn in a neonatal unit is to receive penicillin G benzathine 50,000 units/kg intramuscularly (IM). The newborn weighs 6 lbs (2.7 kg). The dispensed dose is 25,000 units per 1 mL. What should the nurse do? You answered this question Incorrectly 1. Administer the drug intravenously (IV) since a large volume is required. 2. Choose three injection sites and give the medication as prescribed. 3. Consult with the pharmacy for a different medication concentration. 4. Read the available drug information to determine how to administer the medication.

RationaleStrategies 3. Correct: The nurse must consult with the pharmacy to receive further instructions. The dose is greater than the allowed volume to be given IM, which warrants clarification by the pharmacy. 1. Incorrect: Since the drug is prescribed IM, the route should not be changed to IV administration because this violates the prescription as written. 2. Incorrect: The dose is greater than the allowed volume to be given IM, which warrants clarification by the pharmacy. You want to avoid having to give three injections. 4. Incorrect: The concern is not drug information or administration; it is the concentration, which can only be provided by the pharmacy.

A home care nurse is preparing to perform venipuncture to draw blood. As the nurse gathers supplies, the client begins to experience palpitations, trembling, nausea, shortness of breath and a feeling of losing control. What should be the nurse's first action? You answered this question Incorrectly 1. Hug the client to provide support. 2. Take the client to the emergency department for sedation. 3. Decrease stimuli in the room. 4. Teach the client deep breathing exercises.

RationaleStrategies 3. Correct: The nurse should remain calm and quiet by the client. A stimulating environment may increase the client's level of anxiety. 1. Incorrect: Hugging a client is moving into the client;s personal space. Hugging may confine the person and intensify feelings. The nurse should use touch cautiously. 2. Incorrect: Panic attacks usually last minutes, rarely longer. The client is not exhibiting symptoms at this time that would warrant administration of a sedative. 4. Incorrect: This is good; however, you need to wait until the panic attack is over. The client must be ready to learn prior to initiating teaching. The nurse should wait until the symptoms resolve for learning to occur.

A nurse has received the following arterial blood gas results on a client with a post bowel resection: pH 7.48; PCO2 30; HCO3 24. Which acid/base imbalance is the client experiencing? You answered this question Correctly 1. Metabolic alkalosis 2. Metabolic acidosis 3. Respiratory alkalosis 4. Respiratory acidosis

RationaleStrategies 3. Correct: The pH is high, indicating alkalosis. The PCO2 is low, which coordinates with a high pH, indicating a lung problem. This client is in respiratory alkalosis, not acidosis. The bicarbonate is normal. 1. Incorrect: The bicarbonate is normal. In metabolic alkalosis, the bicarbonate will be increased. A pH of 7.48 is an alkalosis. 2. Incorrect: Normal pH is 7.35-7.45. Anything less than 7.35 is an acidosis. Anything greater than 7.45 is an alkalosis. 4. Incorrect: The pH is high, indicating alkalosis. The PCO2 is low, which coordinates with a high pH, indicating a lung problem: Respiratory alkalosis, not acidosis. The bicarbonate is normal.

The nurse is instructing a client on achieving relaxation using deep breathing exercises. Which statement by the client indicates to the nurse that further teaching is necessary? You answered this question Correctly 1. "I can perform deep breathing exercises anywhere and at any time that I feel tension and anxiety." 2. "I should sit or lie in a comfortable position, making sure my back is straight." 3. "I will inhale slowly and deeply through my mouth focusing on my chest expansion." 4. "When I have inhaled in as much as possible, I will hold my breath for a few seconds before exhaling."

RationaleStrategies 3. Correct: The proper method is to inhale slowly and deeply through the nose, allowing the abdomen to expand. The chest should be moving only slightly. 1. Incorrect: This statement demonstrates successful teaching. To relieve anxiety, deep breathing exercises can be initiated as needed. Inhaling slowly and deeply through the nose can be performed anytime, and no additional equipment is needed. 2. Incorrect: This statement demonstrates successful teaching. To initiate deep breathing exercises, the client should sit or lie down in a comfortable position. Maintaining a straight back will facilitate breathing deeply into the lungs. 4. Incorrect: This statement demonstrates successful teaching. Holding your breath after inhaling is a technique that assists the client to control their breathing pattern. The client has control over themselves by repeating the deep breathing exercise.

Blood and urine samples are sent to the laboratory for a client who has had a spinal cord injury. After reviewing these results, the nurse would expect which finding? Exhibit You answered this question Correctly 1. Gross hematuria 2. Septicemia 3. Urinary tract infection 4. Anemia exhibit: Complete Blood Count RBCs 5 million/mm3 (5 X 106 /mm3)​ (5 X 1012 /L​) WBCs 5,000 (5 X 103/mm3) ​(5 X 109 /L​) Urinalysis RBCs 2 to 3/hpf WBCs greater than 5/hpf.

RationaleStrategies 3. Correct: The urinalysis results of red blood cells (RBC) of 2/hpf or greater and urine white blood cells (WBC) of greater than 4/hpf indicate a urinary tract infection (UTI). 1. Incorrect: The urinalysis results of 2 to 3/hpf RBCs is not indicative of gross hematuria 2. Incorrect: The blood WBCs are normal. In septicemia, the blood WBCs are elevated. 4. Incorrect: Blood RBCs of 5 million/mm3 (5 x 106 ​/ mm3) (5 x 1012/ mm3) is a normal finding.

A client develops pernicious anemia after a Billroth II procedure and is to receive vitamin B12 intramuscularly. What should the nurse include in discharge instructions? You answered this question Incorrectly 1. B12 can be stored in a lighted area. 2. The B12 injections will be stopped when symptoms disappear. 3. The B12 injections will be continued for the client's life. 4. Vitamin B12 will be taken by mouth once the maintenance dose is determined.

RationaleStrategies 3. Correct: With pernicious anemia, the client lacks the intrinsic factor. Without the intrinsic factor, B12 cannot be absorbed. The client will require B12 shots throughout the lifespan. 1. Incorrect: B12 should be protected from the light. 2. Incorrect: Cannot be stopped once symptoms disappear due to lack of intrinsic factor. Must be continued throughout the lifespan. 4. Incorrect: B12 cannot be administered orally. The client lacks the intrinsic factor, therefore B12 cannot be absorbed in the GI tract. B12 must be given by injection.

Which intervention can the nurse safely delegate to an unlicensed assistive personnel (UAP)? You answered this question Incorrectly 1. Irrigate a colostomy in a client who is 2 days postoperative. 2. Remove a fecal impaction in a client. 3. Apply a condom catheter to an incontinent client. 4. Insert a urinary catheter to obtain a urine sample.

RationaleStrategies 3. Correct: With proper instruction a UAP may be delegated to apply a condom catheter. This is not an invasive procedure. 1. Incorrect: The care of a new postoperative colostomy should not be assigned to the UAP. This is an invasive procedure and assessments of the stoma, incision, and skin are essential. The RN can not delegate assessment to a UAP. 2. Incorrect: Removal of a fecal impaction is an invasive procedure. Because the bowel mucosa can be injured during this procedure, it is not appropriate to delegate this to the UAP. 4. Incorrect: Insertion of an indwelling urinary catheter is an invasive sterile procedure. The nurse must have knowledge of detailed anatomy and sterile technique. The UAP should not be delegated to perform this procedure.

A charge nurse is planning care for several clients on the unit. Which activities can the nurse safely delegate to an unlicensed assistive personnel (UAP)? You answered this question Incorrectly 1. Administer a nebulizer treatment to a client diagnosed with pneumonia. 2. Obtain vital signs on a postoperative client who required naloxone 5 minutes ago. 3. Report a urinary output (UOP) less than 50 ml/hr on a post-op client. 4. Assist a client with obtaining a clean catch urine sample. 5. Remove an indwelling urinary catheter from a client.

RationaleStrategies 3., & 4. Correct: A UAP can report the amount of UOP but cannot interpret it. A clean catch urine sample is a noninvasive procedure. The UAP can assist the client to obtain the clean catch urinary sample. Both activities are the right person and right task of delegation. 1. Incorrect: A UAP cannot administer medications. This is the wrong task for an UAP. 2. Incorrect: The client received naloxone to reverse the action of an opioid medication. A UAP should not be assigned to obtain vital signs on an unstable client. This is the wrong person to perform removal of an indwelling urinary catheter. 5. Incorrect: A UAP cannot remove an indwelling urinary catheter.

The nurse is teaching a newly diagnosed diabetic about proper foot care. Which statements by the nurse are correct? You answered this question Incorrectly 1. Cut the toenails in a rounded fashion. 2. Wash the feet with warm water and betadine. 3. Wear appropriate fitting shoes at all times. 4. View the bottom of the feet daily. 5. Protect feet from hot and cold.

RationaleStrategies 3., 4., & 5. Correct: Shoes should be worn at all times to prevent injury. The client may step on something and not know that the foot has been injured. Inspection should be done daily, since many diabetics cannot feel if their feet have been injured. Feet may not be sensitive to hot and cold, which could cause injury. 1. Incorrect: Toe nails should be cut straight across to avoid an ingrown toenail. Additionally any skin cuts on the toes may result in infection. 2. Incorrect: Do not put harsh chemicals, such as betadine, on the feet. Betadine will dry the skin which may lead to cracks in the skin. This creates potential portals for infection to occur.

Which actions by the nurse, working in the recovery room, should be performed to prevent injury from a needle stick? You answered this question Correctly 1. Recap the needle after use to prevent injury. 2. Reinsert the sylet if it becomes loose in the vascular assess device. 3. After drawing up saline to flush an intravenous (IV) line, place the syringe in a pocket to prevent possible injury. 4. Replace the puncture resistant biohazard container when three-quarters full. 5. Use "needleless" devices whenever possible.

RationaleStrategies 4. & 5. Correct: Puncture resistant biohazard containers should be replaced when three-quarters full to prevent hand injury when disposing of sharps. Use of "needleless" devices reduces the risk of needle stick injuries. 1. Incorrect: For safety precautions and transmissions of infection, needles should never be recapped due to the possibility of injury while recapping. 2. Incorrect: Reinserting the stylet may cause injury to the nurse and client. 3. Incorrect: For safety precautions of the nurse or another person, a needle should never be placed in a pocket. The cap could come off and stick someone.

The nurse is caring for a client with hypothyroidism. Which dietary consideration is most important for the nurse to teach this client? You answered this question Correctly 1. Increase carbohydrate intake. 2. Decrease fluid intake. 3. Avoid shellfish. 4. Increase fiber.

RationaleStrategies 4. Correct: A symptom of hypothyroidism is constipation due to the decreased mobility of the intestinal tract. Client's with hypothyroidism should increase their dietary fiber to prevent constipation. 1. Incorrect: No, they need fewer calories, not more. Their metabolism is slowed. A client with hypothyroidism may gain weight due to decreased metabolism. The client should decrease their intake of carbohydrates. 2. Incorrect: To decrease constipation the client should increase fluid intake. When the client is hydrated the stool will be softer. 3. Incorrect: Avoiding shellfish is not a consideration unless there is an iodine allergy.

Which client must the nurse assign to a private room? You answered this question Correctly 1. Primiparous client who delivered twins at 28 weeks gestation two days ago 2. Postpartum client on IV Ampicillin and Gentamicin for chorioamnionitis 3. Postpartum client whose 2 hour old infant is being worked up for sepsis 4. Postpartum client 32 hours after delivery with a temperature of 101º F (38.05 ° C)

RationaleStrategies 4. Correct: A temperature of 100.5° F (38.05° C) or greater in a client more than 24 hours postpartum is likely an indication of infection. This client should be kept separate from other mothers and babies. 1. Incorrect: The preterm twins are in the NICU and not in their mother's room (a client with term twins would need a private room because of space considerations). 2. Incorrect: Chorioamnionitis is not contagious. 3. Incorrect: The infant may have an infection and will remain in the NICU. The mother is not infected.

The nurse will be admitting a client from the operating room following a left total pneumonectomy for adenocarcinoma. Which type of chest drainage should the nurse anticipate that the client will have? You answered this question Incorrectly 1. Bilateral chest tubes. 2. One chest tube on the operative side 3. Two chest tubes on the operative side 4. No chest drainage will be necessary.

RationaleStrategies 4. Correct: A total pneumonectomy means the excision of the entire lung. A drainage tube is not inserted, since the fluid and air must accumulate in the thoracic space. This is to prevent mediastinal shift to the left. 1. Incorrect: A total pneumonectomy is removal of entire lung. The presence of fluid and air in the left thoracic cavity decreases the incidence of left mediastinal shift. 2. Incorrect: The entire lung is removed. The left thoracic space should fill with fluid and air to prevent mediastinal shift. The insertion of a chest tube is not warranted. 3. Incorrect: Entire lung is removed and no chest drainage is needed. The insertion of two chest tubes is not required. Fluid and air should fill the left pleural space to reduce the risk of a mediastinal shift.

A client has returned to the burn unit after an escharotomy of the forearm. What is the priority nursing intervention? You answered this question Correctly 1. Roll sterile q-tip over the wound 2. Elevate the affected arm 3. Ask the client to rate pain level 4. Assess bilateral radial pulses

RationaleStrategies 4. Correct: An escharotomy is an incision of the eschar of a burned arm to decrease the tension in the proximal tissue. This will result in increased circulation to the proximal tissue. The assessment of bilateral radial pulses needs to be compared for adequate circulation. 1. Incorrect: This is an intervention for a forearm with a graft. 2. Incorrect: After an escharotomy elevating the affected arm is an appropriate intervention. The evaluation of the circulation in the arm should be performed first. 3. Incorrect: Pain is also an indicator of adequate circulation. The assessment of distal pulses first will provide a more concise assessment of the circulation in both arms.

A client has an order for two units of packed red blood cells (PRBCs) to be administered. The current IV prescribed is D5LR with 20 mEq KCL at 125 mL/hr infusing through a 22 gauge needle to the left hand. What action should the nurse take? You answered this question Correctly 1. Piggyback the PRBCs to the current IV fluid at the lowest port on the tubing. 2. Change the current IV fluid to NS so the blood can infuse through the IV tubing. 3. Disconnect the current IV fluid and connect NS with a y-tubing blood administration set. 4. Start another IV with an 18 gauge needle to the right arm.

RationaleStrategies 4. Correct: Blood should be administered through a large bore IV needle such as an 18 gauge, but no smaller than a 20 gauge. Smaller needles can cause the PRBCs to lyse. 1. Incorrect: The PRBCs must be administered through a y-tubing blood administration set that has a filter. Do not infuse through a normal IV tubing. The current IV that is infusing has 20 mEq KCL added. PRBCs should not be infused with KCL. Also the current IV was initiated with a 22 gauge needle, and the PRBCs should be infused with a needle no smaller than 20 gauge. 2. Incorrect: The IV needle is too small, and the client has an order for the IV fluids and potassium. It is out of the scope of the RN to change the prescription for the D5LR with 20 mEq KCL. Also the client requires additional KCL. 3. Incorrect: The problem here is that the IV needle is too small.

A nurse is teaching a client about the prescription aripiprazole discmelt. The nurse documents that teaching has been effective when the client makes which statement? You answered this question Incorrectly 1. "If I start to have shakiness and sweating I need to call my primary healthcare provider at once." 2. "I must be certain to take this medication with food to eliminate vomiting." 3. "If I miss a dose of medication, I need to take an extra dose to make up for the missed dose." 4. "I will allow the tablet to dissolve in my mouth."

RationaleStrategies 4. Correct: Discmelt is an orally disintegrating tablet. Since this tablet is formulated to dissolve on the tongue, the tablet should not be swallowed. 1. Incorrect: Hyperglycemia can occur. Signs/symptoms include polydipsia, polyphasia, polyuria. Hyperglycemia is a potential adverse reaction of aripiprazole discmelt. The symptoms listed are indicative of hypoglycemia. 2. Incorrect: Can be taken with or without food. Aripiprazole can be taken with or without food. Taking the medication with food does not increase or decrease side effects. 3. Incorrect: Skip the missed dose if it is almost time for the next scheduled dose. Do not take extra medication to make up for missed dose.

What is most important for the nurse to have at the client's bedside when inserting a large orogastric tube for rapid gastric lavage? You answered this question Incorrectly 1. Emesis basin 2. Portable x-ray machine 3. Oxygen 4. Suction equipment

RationaleStrategies 4. Correct: Insertion of a large orogastric tube designed for rapid lavage often causes gagging and vomiting, so suction equipment should be readily available to reduce the risk of aspiration. Maintaining the client's airway is the priority. 1. Incorrect: You would need an emesis basin because of the chance of vomiting, but suction equipment is the priority due to aspiration. 2. Incorrect: An x-ray is the preferred method to check initial placement, once the tubing is inserted. Suction equipment is the priority when inserting the tube due to risk of aspiration. 3. Incorrect: There are no key words in the question to suggest the client needs oxygen at this time.

A parent tells the clinic nurse, "My child has just been diagnosed with attention-deficit/hyperactivity disorder (ADHD). What will be done to help my child?" How should the nurse best respond to the parent? You answered this question Correctly 1. The primary healthcare provider will want to start your child on a central nervous system (CNS) depressant in order to decrease hyperactivity and improve attention. 2. You will need to admit your child to the psychiatric behavioral unit so that group therapy can be initiated. 3. Children are often placed on central nervous system stimulants that improve behavior associated with ADHD. 4. The standard of care for children with ADHD includes central nervous system stimulants along with behavior and family therapy.

RationaleStrategies 4. Correct: Multimodal treatment of ADHD is the standard of care for children. There is a lot to be gained by supporting medication treatment with appropriate educational, psychosocial, and family interventions. 1. Incorrect: Central nervous system (CNS) depressants are not appropriate therapy for ADHD. Central nervous stimulants are recommended to manage ADHD symptoms. 2. Incorrect: Children with ADHD are not generally hospitalized. Outpatient behavior therapy is recommended to assist the client to substitute positive behaviors with nonproductive behaviors. 3. Incorrect: Central nervous system stimulants are given to children with ADHD. This is a true statement, but the standard of care also includes behavior and family therapy.

A nurse is caring for a client who has developed ventricular fibrillation. Where should the nurse place the conductive electrodes for maximum defibrillation effectiveness? You answered this question Incorrectly 1. The left lower sternum and the right side of the thorax in the midclavicular line. 2. On the right shoulder and the left side of the sternum just below the rib cage. 3. The left upper chest to the left of the sternum and the lower right half of the sternum. 4. Below the right clavicle to the right of the sternum and just below the left nipple.

RationaleStrategies 4. Correct: One electrode should be placed just below the clavicle to the right of the sternum, and the other electrode placement is on the left side just under the left nipple (pectoral area) and in the midaxillary line. 1. Incorrect: The position of these electrodes is not the recommended and acceptable placement for optimal defibrillation. The positioning of the electrodes is not anatomically correct to allow maximum delivery of the current through the heart. 2. Incorrect: The position of these electrodes is not the recommended and acceptable placement for optimal defibrillation. The positioning of the electrodes is not anatomically correct to allow maximum delivery of the current through the heart. 3. Incorrect: The position of these electrodes is not the recommended and acceptable placement for optimal defibrillation. The positioning of the electrodes is not anatomically correct to allow maximum delivery of the current through the heart.

The charge nurse observes a staff nurse caring for a new mother with oral herpes simplex type I. Which action by the nurse indicates that further instruction on transmission of this disease is needed? You answered this question Correctly 1. Instructs the new mother that she should not kiss the newborn. 2. Wears gloves during the perineal and lochia assessment. 3. Washes hands before and after each client contact. 4. States that the newborn may contract herpes from the birth canal.

RationaleStrategies 4. Correct: Oral herpes simplex type I is more often manifested by lesions on the lips or nose (cold sores/fever blisters) and is contagious, but not through the birth canal. Genital herpes type 2 can be transmitted to the newborn during child birth. 1. Incorrect: The newborn can contract herpes simplex 1 through direct skin contact with the lesions or oral secretions such as kissing. This is an appropriate instruction. 2. Incorrect: Wearing gloves during the assessment is not related to the mother's diagnosis of oral herpes simplex type These are standard precautions and are appropriate. 3. Incorrect: Washing hands prior to and after each client contact are standard precautions and are appropriate.

In what position should the nurse place a client post liver biopsy? You answered this question Incorrectly 1. Left Sims' 2. Reverse Trendelenburg 3. Semi-Fowler's 4. Right Lateral Decubitus

RationaleStrategies 4. Correct: Right lateral decubitus is defined as lying on the right side. The client is placed on the right side post liver biopsy to reduce bleeding by compressing the liver capsule to the puncture site. 1. Incorrect: Left Sims' position is left side with knee and thigh drawn upward. Being placed on the left side would not decrease the chances of bleeding at the insertion site. This is the proper position for administration of an enema. 2. Incorrect: Reverse Trendelenburg is when the bed is tilted with head upward. This is an incorrect placement post liver biopsy. This position would not compress the liver capsule to the puncture site. 3. Incorrect: Semi-Fowler's is head of bed approximately 30 degrees and is an incorrect position post liver biopsy. This position would not apply pressure from liver capsule to the puncture site.

The nurse is caring for a client while fluorouracil is being infused. The client reports burning at the intravenous (IV) site. What should the nurse do first? You answered this question Correctly 1. Apply warm compresses. 2. Slow the infusion. 3. Inspect the IV site. 4. Stop the infusion.

RationaleStrategies 4. Correct: Stop the infusion in order to stop the damage now. 1. Incorrect: No, it's burning and this is a sign of the drug in the tissues. Apply an ice pack. 2. Incorrect: No, stop the infusion before the arm becomes necrotic. 3. Incorrect: Trust what the client is telling you, and stop it now before too much damage is done.

A client admitted with a myocardial infarction has developed crackles in bilateral lung bases. Which prescription written by the primary healthcare provider should the nurse complete first? You answered this question Incorrectly 1. Draw blood for arterial blood gases. 2. Place compression hose on legs. 3. Insert indwelling catheter for hourly urinary output. 4. Administer furosemide 20 mg intravenous push (IVP).

RationaleStrategies 4. Correct: The client is developing pulmonary edema or heart failure and needs to be diuresed to remove excess fluid. The question stem tells you that you have prescriptions for these four options so what are you going to do first. All prescriptions are possible but furosemide will fix the problem. 1. Incorrect: You may need to draw these to evaluate the effect of the fluid on oxygenation but this option will not be priority over removing the fluid. 2. Incorrect: Compression hose will help prevent stagnation of blood in the lower extremities to prevent deep vein thrombosis (DVT). Great nursing care but not priority when fluid is developing in the lungs. 3. Incorrect: The indwelling catheter can be inserted after administration of the diuretic. You will be glad you have that indwelling catheter but it is not the prescription that will help with the fluid overload problem.

What is most important for the nurse to do prior to initiating peritoneal dialysis? You answered this question Incorrectly 1. Aspirate for placement. 2. Have the client void. 3. Irrigate the catheter for patency. 4. Warm the dialysate fluid.

RationaleStrategies 4. Correct: The peritoneal fluid is inserted into the abdominal cavity. To promote the exchange of wastes and fluid through the peritoneal membrane, the peritoneal fluid should be warmed. This will promote vasodilation of the capillaries in the peritoneal cavity. 1. Incorrect: The peritoneal catheter should not be aspirated. This would not tell you anything and could irritate the peritoneal membrane. After the cover of the dialysate fluid is removed, the tubing should be connected to the peritoneal catheter. 2. Incorrect: This is not a bad choice, just not the most important. Voiding would make the client more comfortable during the procedure but will not affect the success. 3. Incorrect: It is not necessary to irrigate a peritoneal catheter because you are irrigating with the dialysate.

The nurse is caring for a client with myasthenia gravis. What is essential for the nurse to teach this client regarding treatment? You answered this question Incorrectly 1. Frequent low-calorie snacks. 2. Strict monitoring of intake and output. 3. Use of sweeping gaze when walking. 4. Setting the alarm clock for medication times.

RationaleStrategies 4. Correct: Yes! Medication must be taken on time. Too early can cause weakness and too late can cause extreme weakness to point of paralysis. 1. Incorrect: No, the client needs frequent high calorie snacks. 2. Incorrect: No, this is not a cardiac or renal client. 3. Incorrect: No, this is done when the client has homonymous hemianopsia.

Based on the Parkland formula, the primary healthcare provider has determined that a burn victim needs 9,250 mL of LR intravenously over the first 24 hours. How many milliliters of LR should the nurse administer over the first eight hours? Round answer to the nearest whole number. You answered this question IncorrectlyEnter the answer for the question below.

RationaleStrategies Based on the Parkland formula the fluid resuscitation of a burn victim means that ½ of the fluid should be given within the first 8 hours. For this client 4,625 mL needs to be administered within the first 8 hours. The remaining ½ of the fluid is divided over the remaining 16 hours.

A primary healthcare provider has prescribed the insertion of a nasogastric (NG) tube. In what order should the nurse perform this procedure? Place actions in the correct order. You answered this question IncorrectlyThe Correct Order Elevate head of bed to fowler's position. Measure distal NG tube from nose tip to earlobe to xiphoid process. Lubricate 2-3 inches of distal NG tube. Insert NG tube into unobstructed naris. Advance NG tube upward and backward until resistance is met. Rotate catheter and advance into nasopharynx. Have client swallow ice as NG tube advances into stomach. Secure NG tube. Your Selected Order Elevate head of bed to fowler's position. Measure distal NG tube from nose tip to earlobe to xiphoid process. Lubricate 2-3 inches of distal NG tube. Insert NG tube into unobstructed naris. Rotate catheter and advance into nasopharynx. Advance NG tube upward and backward until resistance is met. Have client swallow ice as NG tube advances into stomach. Secure NG tube.

RationaleStrategies First, elevate the client's head of bed to Fowler's position. Second, measure the distal NG tube from the nose tip to the earlobe to the xiphoid process. Third, lubricate 2-3 inches of the distal NG tube. Fourth, insert the NG tube into unobstructed naris. Fifth, advance NG tube upward and backward until resistance is met. Sixth, rotate catheter and advance into oropharynx. Seventh, have client swallow ice to pass the NG tube into the stomach. Eighth, secure the NG tube. The core issue of the question is knowledge of the insertion procedure for a nasogastric tube. Use nursing knowledge to sequence the steps that the nurse needs to take. Visualize the procedure to aid in answering the question.

A client diagnosed with pancreatitis becomes increasingly restless, confused and has pulled out the NG tube and IV catheter. HR-128/min, BP 96/62. Oxygen saturation = 90%. Skin is cool and clammy to touch. Prioritize the actions that the nurse should take. You answered this question IncorrectlyThe Correct Order Initiate oxygen. Insert another IV line. Obtain blood sugar level. Insert NG tube. Repeat vital sign checks Your Selected Order Repeat vital sign checks Initiate oxygen. Obtain blood sugar level. Insert NG tube. Insert another IV line.

RationaleStrategies First, initiate oxygen. The client is anxious and has tachycardia, signs of hypoxia. The BP is also low, so the client might be bleeding internally. If there is a decreased circulating blood volume then there is less hemoglobin to carry oxygen, so increasing the available oxygen will help the client until the problem is corrected. Second, get the IV started so fluid resuscitation can continue.This increased volume will improve the blood pressure. More volume, more pressure. The IV will also provide a port for needed medications. Third, check the client's blood sugar. Since the pancreas is sick, insulin production can be decreased so glucose can go up. This is next in the priority line of the available options. You have addressed air and circulation, so blood glucose would be next. Fourth, insert the NG tube so that the client can be kept empty and dry and you can prevent aspiration if the client starts vomiting. Last, recheck vital signs to assess effectiveness of your nursing actions.

The nurse is caring for a client in the emergency department. The primary healthcare provider prescribed 1000 mL of D5 ½ NS. The IV is infusing at 25 gtts/min. (Drop factor is 60 gtts/mL). What is the infusion time in hours? Round your answer to the nearest whole number. You answered this question CorrectlyEnter the answer for the question below.

RationaleStrategies Step 1 1000 x 60 = 25 total fluid x drop factor = infusion time x time in minutes 60,000 = 25 x 25x = 60,000 x = 2400 min. divide by 60 = 40 hours


संबंधित स्टडी सेट्स

3.8.W - Lesson: The Digestive System

View Set

NUR 418 Quizzes and PP Questions for Exam 3

View Set

Maternity & Child Practice Exam 6

View Set

Real Estate Principals (Ch. 3 - License Law Quiz)

View Set

NUR 240 PrepU Chapter 38: Assessment of Digestive and Gastrointestinal Function

View Set